You are on page 1of 136

目 录

1. Figure Classification ............................................................................... 1

2. Figure Matrices ..................................................................................... 66


1. Figure Classification
1. Explanation
The correct answer is 5.

The first shape is a big five-sided shape (pentagon) with a small six-sided shape
(hexagon) inside. The second shape is a seven-sided shape (heptagon) with a small
eight-sided shape (octagon) inside. The third shape is a big three-sided shape (triangle)
with a small four-sided shape (square) inside.
Therefore, the correct answer will be a big shape with a smaller shape inside that has
one more side than the big shape.

In the 1st answer choice, the small shape has two less sides than the big shape.
In the 2nd answer choice, the big shape does not have any sides.
In the 3rd answer choice, the small shape has three less sides than the big shape.
In the 4th answer choice, the small shape has one less side than the big shape.

2. Explanation
The correct answer is 1.

In this question all of the edges of the shapes in the three figures are straight. The only
answer choice that is like this is the 1st choice. The rest of the choices have one or
more rounded corners.

Therefore, the 1st choice is the correct answer.

3. Explanation
The correct answer is 5.

In this question the shapes in all three figures are equilateral (this means that all the
sides of each shape are the same length). The only answer choice that is like this is the
5th choice. The rest of the choices have one or more sides that are a different length
than the rest.

Therefore, the 5th choice is the correct answer.

4. Explanation
The correct answer is 3.

In this question the three figures are all shapes that have a green triangle inside on one
of the sides of the shape. The only answer choice that is like this is the 3rd choice.
The 1st, 2nd and 4th choices are incorrect as the triangle is in a corner of the outer
shape and not on one of the sides. The 5th choice is also incorrect as the triangle it
contains is white and not green.

1
Therefore, the 3rd choice is the correct answer.

5. Explanation
The correct answer is 1.

In this question all three figures are shapes that are divided by a horizontal and a
vertical line inside. The only answer choice that is like this is the 1st choice. The rest
of the choices are divided by diagonal lines.

Therefore, the 1st choice is the correct answer.

6. Explanation
The correct answer is 3.

In this question the three figures are all four sided shapes filled with a dotted pattern.
The only answer choice that is like this is the 3rd choice. The 1st, 2nd and 5th choices
are incorrect as they are not four sided. The 2nd and 4th choices are also incorrect as
they are filled with a broken line pattern as opposed to a dotted pattern.

Therefore, the 3rd choice is the correct answer.

7. Explanation
The correct answer is 5.

In this question the three figures are all different colored octagons with a narrow plus
sign in the middle which is filled with various patterns. Notice that the octagon itself
is always a solid color. The only answer choice that is like this is the 5th choice. The
1st choice is incorrect as it has a wide border around the octagon shape but shouldn't.
The 2nd choice is incorrect because the octagon itself is patterned while the plus sign
isn't and it should be the opposite. The 3rd choice is incorrect as it is a square and not
an octagon. Finally, the 4th choice is incorrect as it has a plus sign in the center that is
too large.

Therefore, the 5th choice is the correct answer.

8. Explanation
The correct answer is 3.

All three figures contain a diagonal line that slants upward toward the right. All the
diagonal lines in the question are parallel, meaning they have the same steepness and
point the same way.
The only answer choice that follows this rule is the third choice. The rest of the
options do not contain a similar diagonal line that is parallel to the other diagonal
lines.

2
Therefore, the third choice is the correct answer.

9. Explanation
The correct answer is 5.

All three figures in this question contain circles with a vertical line that runs through
the middle.
In each half of the circle, we see a smaller shape with a diagonal line that
slants upwards and towards the middle of the outer circle.

The only answer choice which follows this rule is the 5th choice.
Therefore, the 5th choice is the correct answer.

10. Explanation
The correct answer is 1.

All three figures in this question contain shapes divided in half.


The only answer choice which follows this rule is the 1st choice.
Therefore, the 1st choice is the correct answer.

11. Explanation
The correct answer is 5.

At first glance, we see that all three figures in this question are shapes that point
upwards and towards the right.
However, there is more than one answer choice that points upwards and towards the
right.
So, which answer is correct?

Let’s look again at the shapes in the top row. What else do they have in common?
When we look closely, we see that all 3 shapes are comprised of straight lines.
The only answer choice which follows this rule is the 5th choice. Since there are
multiple options that point up and to the right, and answer 5 is the only one of the
choices that is composed of only straight lines, we know that option 5 must be the
right answer even though it points to the left. Pointing to the right is not a relevant
factor since it is sufficient that the three shapes above and option 5 are all composed
of only straight lines.

Therefore, the 5th choice is the correct answer.

12. Explanation
The correct answer is 3.

In this question, we can see that:

3
• All three figures include a cylinder, a rectangle and a triangle
• The shapes are either white, dark or light blue
• Each shape and shading appear only once in each figure

Answer choice 1 can be eliminated because it contains a hexagon instead of a


rectangle.
Answer choice 2 can be eliminated because more than one shape is dark.
Answer choice 4 can be eliminated because more than one shape is white.
Answer choice 5 can be eliminated as well since the rectangle has been replaced with
a different shape.

The only answer choice which follows all the rules is the 3rd choice.

Therefore, the 3rd choice is the correct answer.

13. Explanation
The correct answer is 4.

All three figures in this question contain the same shape only rotated: a circle
divided into three sections with a smaller shape in each section.
Therefore, we need to find an answer choice which also contains the same shape, no
matter how it is positioned.

Answer choices 1, 2 and 3 can be eliminated as they do not contain a circle in the
middle section like the figures in the top row.
When we look more closely at answer choice 5, we see that the square and the
triangle have switched places.

Therefore, we are left with the 4th choice, which is the only correct answer.

14. Explanation
The correct answer is 2.

Each of the three figures in this question includes one curved line and
several straight lines.

In answer choice 1 the shape has two curved lines and not one.
The shape in answer choice 3 has two curved lines as well.
The shape in answer choice 4 has no curved lines at all.
The shape in answer choice 5 has only a curved line and no straight lines.
Answer choice 2 is the only one containing straight lines and one curved line.

15. Explanation
The correct answer is 4.

4
Each of the figures in this question has seven sides and a triangle on the inside.
The only answer choice which follows these rules is the 4th choice.
All of the other answer choices contain shapes with fewer than seven sides, and/or the
inner shape is not a triangle.

Therefore, the 4th choice is the correct answer

16. Explanation
The correct answer is 1.

Each of the three figures in this question contains only one dark triangle and only
one white triangle that are opposite each other.
The only answer choice that follows this rule is the 1st choice.
The 2nd and 3rd answer choices include two dark triangles and no white triangles.
The 4th answer choice does not include a dark triangle.
The 5th answer choice does not include a white triangle.

Therefore, the 1st choice is the correct answer.

17. Explanation
The correct answer is 4.

In this question all three figures are:


• Triangles,
• Which are divided into three smaller triangles…
• …that all meet at a single point.

Furthermore, in all three figures, if the figure is rotated in a way that the meeting point
is at the top of the figure:
• The small triangle in the middle is white,
• The small triangle on the right is dark, and -
• The small triangle on the left is striped.

The only answer choice which follows all of these rules is the 4th choice.
Notice that in the 5th answer choice, if the figure is rotated in a way that the meeting
point is at the top of the figure (and not at the bottom as it is in the answer) the dark
small triangle will be on the left, and the striped small triangle will be on the right.
That is the opposite order of what is shown in the series. Thus, the 5th answer choice
is incorrect.
Therefore, the 4th choice is the correct answer

18. Explanation
The correct answer is 4.

5
All three figures contain a house with:
• A tiled roof,
• One door and –
• Two rectangular windows.

The only answer choice which follows this rule is the 4th choice.

Answer choice 1 has three doors and one window.


Answer choice 2 has only one window and a roof which is not tiled.
Answer choice 3 also has a roof which is not tiled.
Answer choice 5 has circular windows.

Therefore, the 4th choice is the correct answer.

19. Explanation
The correct answer is 5.

All three figures in this question contain a frame with two suns and five clouds.
The only answer choice which follows this rule is the 5th choice.
Answer choice 1 is incorrect because it contains three suns and three clouds.
Answer choice 2 is incorrect because it contains one sun and five clouds.
Answer choice 3 is incorrect because it contains two suns and six clouds.
Answer choice 4 is incorrect because it contains three suns and five clouds.

Therefore, the 5th choice is the correct answer.

20. Explanation
The correct answer is 4.

In this question, we can see the following:


All three figures are shapes containing one diagonal inner line.
The inner line divides each shape into two unequal parts.
All three shapes contain a cloud, which is located above the inner line.

In the 4th answer choice, there is a shape with a diagonal inner line, dividing that
shape into two unequal parts. Moreover, this shape contains
a cloud located above the inner line.

The 1st and 2nd answer choices are incorrect since in both cases the inner line is not
diagonal and divides the shape into two equal parts.
The 3rd answer choice is incorrect as the cloud is located below the inner line while it
should be above the line.
The 5th answer choice is incorrect as it contains two inner lines instead of one.

6
21. Explanation
The correct answer is 2.

In this question, we can see that:


 All three figures contain a shape with two inner lines that meet at a single
point.
 One of the inner lines is straight, while the other is curved.

The 1st answer choice is incorrect as the two inner lines are straight.
The 3rd answer choice is incorrect as this shape contains three inner lines.
The 4th answer choice is incorrect as the two inner lines inside the shape do not meet
at any point.
The 5th answer choice is incorrect as the two inner lines are curved.

In the 2nd answer choice, we can see a shape containing two inner lines that meet at
a single point. One of the lines is straight and the other is curved.

Therefore, the 2nd answer choice is the correct answer.

22. Explanation
The correct answer is 5.

In this question, we can see that all three figures in the top row consist of three
identical shapes that share a common area:

7
Similarly, in the 5th answer choice, we can see that all three circles share a common
area:

The rest of the answer choices do not contain an area that is shared by all three
shapes.

Therefore, the 5th answer choice is the correct answer.

23. Explanation
The correct answer is 2.

In this question, we can see that:


 All three figures contain a shape with an inner arrow pointing to one of the
shape’s corners.
 In each of the figures, one of the shapes (either the inner arrow or the outer
shape) is white, while the other shape is dark blue.

In the 2nd answer choice, we can see a shape (triangle) with an inner arrow pointing
to one of its corners. In addition, the triangle is dark blue, while the arrow inside it
is white.

Therefore, the 2nd answer choice is the correct answer.


24. Explanation
The correct answer is 3.
All three figures in this question are divided into two equal halves. In addition, the
separating line is a diagonal line slanted up to the left.
The third figure is divided into two equal halves, and the separating line is a diagonal
line which slants up to the left. Thus, this is the correct answer.
The first answer choice is incorrect as the separating line slants up to the right.

8
The second answer choice is incorrect as the separating line is horizontal when it
should be diagonal.
The fourth answer choice is incorrect as the separating line slants up to the right.
The fifth answer choice is incorrect as the figure is not divided into two equal halves.

25. Explanation
The correct answer is 5.
In this question, all of the three figures above are circles divided into four parts. Each
one of the circles contains one smiley face and one star. The smiley face and the star
are located opposite to each other.
The fifth figure contains one smiley face and one star that are located opposite to each
other. Thus, this is the correct answer.
The first answer choice is incorrect as it does not contain a star.
The second and fourth answer choices are incorrect as the smiley face and the star are
located next to each other instead of opposite each other.
The third answer choice is incorrect as it does not contain a smiley face.

26. Explanation
The correct answer is 1.

All three figures in this question are comprised of:

 An outer shape made up of curved lines only.


 Two identical black inner shapes made up of straight lines only.
The only answer choice which follows these rules is the 1st one. The outer shape is
made up of curved lines only, and the two black inner shapes (hexagons) are made
up of straight lines only.
Therefore, the 1st choice is the correct answer.

27. Explanation
The correct answer is 2.
In this question, all three figures above consist of a big shape in the middle, with
the same two smaller shapes, one on the right and one on the left. In addition, notice
that all the figures are aligned, and that the smaller shapes are in the same direction as
the big shape.
The second figure consists of a big pentagon in the middle with two smaller
pentagons on its right and left. Notice that the small pentagons are in the same
direction as the big pentagon, and that the whole figure is aligned. Thus, this is the
correct answer.
The first answer choice is incorrect as the two smaller rectangles are located above
and beneath the big rectangle, instead of being located on its sides.
The third answer choice is incorrect as the small pentagons are pointing downwards
while the big pentagon is pointing upwards, when instead they all should be on the
same direction.

9
The fourth answer choice is incorrect as the figure is not aligned but rather slants to
the right.
The fifth answer choice is incorrect as the smaller shapes (rectangles) are not the same
as the big shape (the pentagon).

28. Explanation
The correct answer is 1.
In this question, you can see that the three figures above consist of three shapes each,
which are set in a diagonal manner: a circle, a trapezoid, and a star. In addition, in
each figure, the shapes (the circle, the trapezoid, and the star) are either dark
blue, white, or checkered. Notice that there is no match between a specific shape and
a specific color, and that no two shapes in the same figure have the same filling.

The first figure consists of the three correct shapes: the circle, the trapezoid, and the
star. These shapes are correctly set diagonally. In addition, these shapes have the
correct colors: white (the trapezoid), dark blue (the circle), and checkered (the star).
Thus, this is the correct answer.
The second and third answer choices are incorrect as although they consist of the
correct shapes and have the correct colors, the shapes in them are not set diagonally,
and therefore they can be eliminated.
The fourth and fifth answer choices are incorrect as although they have the correct
shapes which are correctly set diagonally, they do not have the correct colors: the
fourth answer choice is not white, and the fifth answer choice is not checkered.
Therefore, these answer choices can be eliminated.

29. Explanation
The correct answer is 3.
In this question, the three figures above have straight sides and are filled in
with diagonal lines that slant up to the right.
The third answer choice follows these conditions: it is a shape that consists of only
straight sides, and is filled in with diagonal lines slanted up to the right. Therefore,
this is the correct answer.

The first, fourth, and fifth answer choices are incorrect as they all consist of curved
lines as well as straight lines.
The second answer choice is incorrect as although it consists of straight lines only, it
is filled in with diagonal lines slanted up to the left instead of being slanted up to the
right.
Tip: If you find it difficult to infer the rule connecting the three figures above, it
might be useful to look at the answer choices to see if there are any unusual figures. In
this question, you can see that only answer choices 2 and 3 have straight lines, so you
can check if this is the correct rule.

10
30. Explanation
The correct answer is 4.
In this question, the three figures above all contain a diagonal line that connects one
of the shape’s corners with one of the shape’s sides. Notice that the line does
not connect two corners.
The shape in the fourth answer choice contains a diagonal line that stretches from its
bottom left corner to the middle of its upper right side. Therefore, this is the correct
answer.
The first answer choice is incorrect as the line in this shape connects two sides and
does not reach a corner.
The second answer choice is incorrect as the line in this shape connects only corners
and does not reach one of the shape’s sides.
The third answer choice is incorrect as there are two lines in this shape while there
should be only one.
The fifth answer choice is incorrect as the line in this shape connects two sides and
does not reach a corner.

31. Explanation
The correct answer is 3.
In this question, the three figures above all contain two inner lines
that intersect inside the shape. Notice that the inner lines divide each figure
into four parts (not necessarily equal).
The third answer choice is a pentagon which contains two inner lines that intersect
inside it. In addition, these lines divide the pentagon into four parts. Therefore, this is
the correct answer.
The first and the second answer choices are incorrect as the two inner lines in these
figures do not intersect.
The fourth answer choice is incorrect as this figure contains three inner lines instead
of two.
The fifth answer choice is incorrect as although this figure is divided into four parts, it
contains three inner lines instead of two.

32. Explanation
The correct answer is 1.
In this question, the three figures above are divided into two parts: a white part and
a checkered part. In addition, within the white part, there is a small white
shape which is the same as the outer figure.
The first answer choice is a pentagon divided into a white part and a checkered part.
Within the white part there is a small pentagon, which is the same as the outer figure.
Therefore, this is the correct answer.
The second answer choice is incorrect as the small shape is the same as the shape of
the white part (a trapezoid), while it should be the same as the outer figure (a
hexagon).
The third answer choice is incorrect as the small shape is located within the checkered

11
part instead of the white part.
The fourth answer choice is incorrect as the small shape (a triangle) is checkered
instead of being white. In addition, the outer figure (except for the inner small triangle)
is white, while it should be both white and checkered.
The fifth answer choice is incorrect as the small shape (a triangle) is different than the
outer figure (a parallelogram).

33. Explanation
The correct answer is 5.
In this question, we can see that:
 The three figures are circles containing two inner shapes, one to each side of
an inner line that cuts the circles in half.
 One of the inner shapes is white, while the other inner shape is black.
 In each figure, the two inner shapes have a different number of sides.
The 1st and 3rd answer choices are incorrect as the two inner shapes in these choices
have the same number of sides.
The 2nd and 4th answer choices are incorrect as the two inner shapes in these choices
are the same color.
The 5th answer choice is a circle containing one black shape and one white shape. In
addition, these two shapes have a different number of sides (pentagon with five
sides and triangle with three sides).
Therefore, the 5th answer choice is the correct answer.

34. Explanation
The correct answer is 4.

In this question the three figures are all shapes that have perfect squares in the center.
The only answer choice that is like this is the 4th choice. The 2nd, 3rd and 5th choices
are incorrect as they contain inner shapes that are not square and have the wrong
number of sides. The 2nd choice is also incorrect as it contains a four sided shape that
is not a square but a diamond.

Therefore, the 4th choice is the correct answer.

35. Explanation
The correct answer is 4.

In this question all three figures are shapes with inverted corners (some of the shape's
corners point into the shape). The only answer choice that is like this is the 4th choice.
The rest of the choices only contain corners that point outward.

Therefore, the 4th choice is the correct answer.

12
36. Explanation
The correct answer is 1.

In this question the three figures all contain a shape with curved sides inside a shape
that only has straight sides. Notice that the inner and outer shapes are always different
colors. The only answer choice that is like this is the 1st choice. The 2nd choice is
incorrect because the inner shape is straight and outer shape is curved while the
opposite should be true. The 3rd choice is incorrect as both the inner and outer shapes
are the same color. The 4th choice is incorrect as both of the shapes have curved sides.
Finally, the 5th choice is incorrect as the both of the shapes have only straight sides.

Therefore, the 1st choice is the correct answer.

37. Explanation
The correct answer is 5.

In this question all three figures are shapes that have no inverted corners (all of the
shapes corners point outward). The only answer choice that is like this is the 5th
choice. The rest of the choices contain corners that point inverted corners (that
point into the shape).
Alternatively, another pattern this question presents is a series of shapes in which each
shape has one side less than the one before. The first shape is a green hexagon (6
sides), followed by a purple pentagon (5 sides) and an orange quadrilateral (3 sides).
The next shape in the series, therefore, would be a triangle (3 sides).
Therefore, the 5th choice is the correct answer.

38. Explanation
The correct answer is 2.

In this question the three figures are all purple octagons with a narrow plus sign in the
middle which is filled with various patterns. Notice that the octagon itself is always
solid purple. The only answer choice that is like this is the 2nd choice. The 1st choice
is incorrect as it has a plus sign in the center that is too large. The 3rd choice is
incorrect as it is a square and not an octagon. The 4th choice is incorrect because the
octagon itself is patterned while the plus sign isn't and it should be the opposite.
Finally, the 5th choice is incorrect as it has a wide border around the octagon shape
but shouldn't.

Therefore, the 2nd choice is the correct answer.

39. Explanation
The correct answer is 4.

In this question all three figures are orange circles each of which contains the

13
following three shapes: a yellow diamond, triangle and circle. The only answer choice
that is like this is the 4th choice. The 1st choice is incorrect as it contains two
diamonds and no triangle. The 2nd and 3rd choices are incorrect as they contain
trapezoids instead of triangles. Finally, the 5th choice is incorrect as it contains four
shapes instead of three (a trapezoid in addition to all of the other shapes).

Therefore, the 4th choice is the correct answer.

40. Explanation
The correct answer is 2.

In this question all three figures are triangles each of which contains the following
three shapes: a tilted square, a triangle and a circle. The only answer choice that is like
this is the 2nd choice. The 1st, 3rd and 4th choices are incorrect as they contain
diamonds instead of squares, and the 5th choice is incorrect as it contains two
triangles but no square.

Therefore, the 2nd choice is the correct answer.

41. Explanation
The correct answer is 2.

The three figures in this question include a bigger shape in the middle with smaller
shapes on top and at the bottom. The smaller shapes are a smaller version of the
upper half of the bigger shape.

This eliminates answer choice 1, as the smaller shapes are different from each other
and from the bigger shape.
Answer choice 5 is also eliminated, as the smaller shapes (pentagons) are the same as
the entire bigger shape (a pentagon), not the upper half.
Answer choice 3 is also wrong, as the smaller shapes above and below the bigger
shape show the entire bigger shape, only rotated.

We are now left with answer choices 2 and 4.


If you look closely, you will see that in answer choice 4, the smaller shape at the
bottom is turned upside down, as if it is the lower half of the bigger shape. Since it
must be the upper half, this is incorrect.

The 2nd choice is the only one left that follows these rules, so it is the correct answer.

14
42. Explanation
The correct answer is 3.

All three figures in this question contain rectangles that are divided into four, with a
circle and a rhombus inside.

This eliminates answer choices 2, 4, and 5, as they contain shapes that are not a circle
and rhombus.

How do we choose between answer choices 1 and 3?


Look again at the three figures in the top row - what else do they have in common?
We can see that in all three figures, the circle and the rhombus appear next to each
other.
The only answer choice which follows this rule is the 3rd choice.
Therefore, the 3rd choice is the correct answer.

43. Explanation

The correct answer is 5.

All three figures in this question contain shapes divided (approximately) into
one-third (1/3) and two-thirds (2/3) parts.
The only answer choice which follows this rule is the 5th choice.
The 1st, 2nd, and 3rd choices are all divided evenly into halves, so they are incorrect.
The 4th choice is clearly not divided in the same way as the shapes in the question, as
the smaller part is much smaller and the larger part is much larger.
Therefore, the 5th choice is the correct answer.

44. Explanation
The correct answer is 4.

In this question all three figures have:


• A rectangle at the bottom,
• A circle in the middle, and –
• A triangle at the top.
The only answer choice which follows these rules is the 4th choice.

Therefore, the 4th choice is the correct answer.

45. Explanation
The correct answer is 5.

Each of the three figures in this question has two corners. A corner is where two
straight lines meet.

15
In answer choice 1 the shape has three such corners.

The shape in answer choice 2 has five such corners.

Answer choice 3 has no corners at which two straight lines meet.


Answer choice 4 has three such corners.

Therefore, the 5th choice is the correct answer.

16
46. Explanation
The correct answer is 5.

Each of the figures in this question has six sides and an oval shape on the inside.

The only answer choice which follows these rules is the 5th choice.

All of the other answer choices contain shapes with fewer than six sides, and the
shapes inside them are not oval.

Therefore, the 5th choice is the correct answer.

47. Explanation
The correct answer is 3.

All three figures in this question contain:


• An outer shape which is a square
• An inner shape whose top part is textured, and
• A background whose bottom part is textured

The only answer choice which follows all of these rules is the third choice.

The first choice is incorrect because the textured parts are the bottom of the inner
shape and the top of the background instead of the other way around.
The second choice is incorrect because the outer shape is a circle instead of a square.
The fourth choice is incorrect because both parts of the inner shape are textured, and
the background has no textured part at all.
The fifth choice is incorrect because there is no inner shape, and the bottom part of
the background has 2 different textures. (an alternative explanation is that the inner
shape is a rectangle and it is fully textured, instead of just the top half.)
Therefore, the 3rd choice is the correct answer.

48. Explanation
The correct answer is 2.

All three figures in this question contain:


• An outer shape which is a circle,
• A 5-sided shape (pentagon) inside the circle, and –
• A smaller 4-sided shape (quadrilateral) inside the pentagon

The only answer choice which follows these rules is the 2nd choice.
Therefore, the 2nd choice is the correct answer.

17
49. Explanation
The correct answer is 5.

In this question all three figures are shapes with three squares, some appearing in 3D,
with one shape in each square.
The squares in all three figures contain a sun, a heart and a banner.

Answer choice 1 is a shaped like a can and does not contain three squares.
Answer choice 2 does contain three squares, but the banner does not look like the one
in the figures above.
Answer choice 3 does not contain three squares with a shape in each one.
Answer choice 4 contains an octagon and not three squares.

Therefore, the 5th choice is the correct answer.

50. Explanation
The correct answer is 1.

Each of the three figures in this question includes:


• Two curly brackets,
• A circle,
• A triangle,
• A hexagon, and –
• A pentagon

This eliminates answer choice 3, as it contains only one curly bracket.


Answer choice 4 can also be eliminated since it contains square brackets and not curly
ones.
Answer choice 2 can be eliminated because it contains two triangles and two circles.
Answer choice 5 can be eliminated because it contains two squares.

We are left with answer choice 1, which is the correct answer.

51. Explanation
The correct answer is 3.

In this question, we can see that:


 All three figures consist of an outer shape with a smaller inner shape. The
smaller inner shape is similar to the outer shape.
 All shapes are comprised of straight lines only.
 The outer shape is colored with light blue that fades out from bottom to top.

The 1st choice is incorrect as it is comprised of curved lines.

18
The 2nd choice is incorrect as it is also comprised of curved lines.
The 4th choice is incorrect as the outer shape is colored with light blue that fades out
from right to left.
The 5th choice is incorrect as the inner shape is colored instead of the outer shape.

Therefore, the 3rd choice is the correct answer.

52. Explanation
The correct answer is 5.

In this question, we can see that:


All three figures are comprised of two shapes.
One of the shapes is located in the front and the other shape in the back.
The shape in the back has one side more than the shape in the front.

We can see that in the 5th answer choice, the shape in the back has six sides, while
the shape in the front has five sides. This means that the shape in the back has one
side more than the shape in the front.

Therefore, the 5th answer choice is the correct answer.

53. Explanation
The correct answer is 3.

In this question, if we straighten the figures in the top row, we can see that all three of
them contain the same shapes, in the same order. The shapes follow this
order (from left to right): a division sign, an arrow, a white triangle, a dark heart, and
a white circle.

The 1st choice is incorrect as, from left to right, the arrow should come before the
triangle and not after it.
The 2nd choice is incorrect as it contains a white heart; the heart should be dark.
The 4th choice is incorrect as, from left to right, the first shape should be the division
sign and not the circle.
The 5th choice is incorrect as it contains a dark triangle and a white heart; the triangle
should be white and the heart should be dark.

The 3rd answer choice contains the aforementioned shapes in the exact same order,
from left to right, as the figures in the top row.

Therefore, the 3rd answer choice is the correct answer.

19
54. Explanation
The correct answer is 1.

In this question, we can see that:


 The three figures are squares divided into nine smaller parts.
 In each figure, there are three Xs and one circle.
 In each figure, the three Xs are organized in a sequence of a straight or a
diagonal line.
 In each figure, the circle is located in one of the bigger square’s corners.

In the 1st answer choice, we can see a square divided into nine smaller parts. This
square contains three Xs and one circle. We can see that the Xs are organized in
a sequence of a straight line and that the circle is located in one of the bigger
square’s corners.

Therefore, the 1st answer choice is the correct answer.

55. Explanation
The correct answer is 3.

In this question, you can see that:

•Each of the three figures consists of two similar shapes, one on top of the other.
•The top shape is similar to the bottom shape, only rotated 90 degrees clockwise.

In the 3rd answer choice, you can see two similar shapes (trapezoids), one on top of
the other. In addition, the top trapezoid is similar to the bottom one, only rotated 90
degrees clockwise.

Therefore, the 3rd answer choice is the correct answer.

The first answer choice shows two shapes side by side instead of one on top of the
other, and therefore it cannot be correct.

The second answer choice has one shape on top, but it has only been rotated through
45 degrees instead of 90 degrees, so it cannot be correct.

The fourth answer choice shows two shapes, where the top has been rotated through
180 degrees instead of 90 degrees, so it cannot be correct.

In the fifth answer choice, the top shape has been rotated 90 degrees anticlockwise so
the top arrow is pointing west instead of east. Therefore, it cannot be correct.

20
56. Explanation
The correct answer is 5.

In this question, we can see that:


 All three figures are shapes divided into smaller parts.
 In each shape, half of the smaller parts are colored: In the circle and the
diamond, four out of eight parts are colored. In the rectangle, two out of
four parts are colored.

In the 5th answer choice, we can see a hexagon divided into six smaller
parts. Three out of six parts are colored, which means that half of the hexagon’s total
area is colored.

Therefore, the 5th answer choice is the correct answer.

57. Explanation
The correct answer is 5.

The first shape is a big five-sided shape (pentagon) with a small six-sided shape
(hexagon) inside. The second shape is a seven-sided shape (heptagon) with a small
eight-sided shape (octagon) inside. The third shape is a big three-sided shape (triangle)
with a small four-sided shape (square) inside.
Therefore, the correct answer will be a big shape with a smaller shape inside that has
one more side than the big shape.
In the 1st answer choice, the small shape has two less sides than the big shape.
In the 2nd answer choice, the big shape does not have any sides.
In the 3rd answer choice, the small shape has three less sides than the big shape.
In the 4th answer choice, the small shape has one less side than the big shape.

58. Explanation
The correct answer is 1.

In this question all of the edges of the shapes in the three figures are straight. The only
answer choice that is like this is the 1st choice. The rest of the choices have one or
more rounded corners.

Therefore, the 1st choice is the correct answer.

59. Explanation
The correct answer is 5.

In this question the shapes in all three figures are equilateral (this means that all the
sides of each shape are the same length). The only answer choice that is like this is the

21
5th choice. The rest of the choices have one or more sides that are a different length
than the rest.

Therefore, the 5th choice is the correct answer.

60. Explanation
The correct answer is 3.

In this question the three figures are all shapes that have a green triangle inside on one
of the sides of the shape. The only answer choice that is like this is the 3rd choice.
The 1st, 2nd and 4th choices are incorrect as the triangle is in a corner of the outer
shape and not on one of the sides. The 5th choice is also incorrect as the triangle it
contains is white and not green.

Therefore, the 3rd choice is the correct answer.

61. Explanation
The correct answer is 1.

In this question all three figures are shapes that are divided by a horizontal and a
vertical line inside. The only answer choice that is like this is the 1st choice. The rest
of the choices are divided by diagonal lines.

Therefore, the 1st choice is the correct answer.

62. Explanation
The correct answer is 3.

In this question the three figures are all four sided shapes filled with a dotted pattern.
The only answer choice that is like this is the 3rd choice. The 1st, 2nd and 5th choices
are incorrect as they are not four sided. The 2nd and 4th choices are also incorrect as
they are filled with a broken line pattern as opposed to a dotted pattern.

Therefore, the 3rd choice is the correct answer.

63. Explanation
The correct answer is 5.

In this question the three figures are all different colored octagons with a narrow plus
sign in the middle which is filled with various patterns. Notice that the octagon itself
is always a solid color. The only answer choice that is like this is the 5th choice. The
1st choice is incorrect as it has a wide border around the octagon shape but shouldn't.
The 2nd choice is incorrect because the octagon itself is patterned while the plus sign
isn't and it should be the opposite. The 3rd choice is incorrect as it is a square and not

22
an octagon. Finally, the 4th choice is incorrect as it has a plus sign in the center that is
too large.

Therefore, the 5th choice is the correct answer.

64. Explanation
The correct answer is (3).

All three figures contain a diagonal line that slants upward toward the right. All the
diagonal lines in the question are parallel, meaning they have the same steepness and
point the same way.
The only answer choice that follows this rule is the third choice. The rest of the
options do not contain a similar diagonal line that is parallel to the other diagonal
lines.

Therefore, the third choice is the correct answer.

65. Explanation
The correct answer is 5.

All three figures in this question contain circles with a vertical line that runs through
the middle.
In each half of the circle, we see a smaller shape with a diagonal line that
slants upwards and towards the middle of the outer circle.

The only answer choice which follows this rule is the 5th choice.
Therefore, the 5th choice is the correct answer.

66. Explanation
The correct answer is 1.

All three figures in this question contain shapes divided in half.


The only answer choice which follows this rule is the 1st choice.
Therefore, the 1st choice is the correct answer.

67. Explanation
The correct answer is 5.

At first glance, we see that all three figures in this question are shapes that point
upwards and towards the right.
However, there is more than one answer choice that points upwards and towards the
right.
So, which answer is correct?

23
Let’s look again at the shapes in the top row. What else do they have in common?
When we look closely, we see that all 3 shapes are comprised of straight lines.
The only answer choice which follows this rule is the 5th choice. Since there are
multiple options that point up and to the right, and answer 5 is the only one of the
choices that is composed of only straight lines, we know that option 5 must be the
right answer even though it points to the left. Pointing to the right is not a relevant
factor since it is sufficient that the three shapes above and option 5 are all composed
of only straight lines.

Therefore, the 5th choice is the correct answer.

68. Explanation
The correct answer is 3.

In this question, we can see that:


• All three figures include a cylinder, a rectangle and a triangle
• The shapes are either white, dark or light blue
• Each shape and shading appear only once in each figure

Answer choice 1 can be eliminated because it contains a hexagon instead of a


rectangle.
Answer choice 2 can be eliminated because more than one shape is dark.
Answer choice 4 can be eliminated because more than one shape is white.
Answer choice 5 can be eliminated as well since the rectangle has been replaced with
a different shape.

The only answer choice which follows all the rules is the 3rd choice.

Therefore, the 3rd choice is the correct answer.

69. Explanation
The correct answer is 4.

All three figures in this question contain the same shape only rotated: a circle
divided into three sections with a smaller shape in each section.
Therefore, we need to find an answer choice which also contains the same shape, no
matter how it is positioned.

Answer choices 1, 2 and 3 can be eliminated as they do not contain a circle in the
middle section like the figures in the top row.
When we look more closely at answer choice 5, we see that the square and the
triangle have switched places.

Therefore, we are left with the 4th choice, which is the only correct answer.

24
70. Explanation
The correct answer is 2.

Each of the three figures in this question includes one curved line and
several straight lines.

In answer choice 1 the shape has two curved lines and not one.
The shape in answer choice 3 has two curved lines as well.
The shape in answer choice 4 has no curved lines at all.
The shape in answer choice 5 has only a curved line and no straight lines.

Answer choice 2 is the only one containing straight lines and one curved line.

Therefore, the 2nd choice is the correct answer.

71. Explanation
The correct answer is 1.

Each of the three figures in this question contains only one dark triangle and only
one white triangle that are opposite each other.
The only answer choice that follows this rule is the 1st choice.
The 2nd and 3rd answer choices include two dark triangles and no white triangles.
The 4th answer choice does not include a dark triangle.
The 5th answer choice does not include a white triangle.

Therefore, the 1st choice is the correct answer.

72. Explanation
The correct answer is 4.

All three figures contain a house with:


• A tiled roof,
• One door and –
• Two rectangular windows.

The only answer choice which follows this rule is the 4th choice.

Answer choice 1 has three doors and one window.


Answer choice 2 has only one window and a roof which is not tiled.
Answer choice 3 also has a roof which is not tiled.
Answer choice 5 has circular windows.

Therefore, the 4th choice is the correct answer.

25
73. Explanation
The correct answer is 5.

All three figures in this question contain a frame with two suns and five clouds.
The only answer choice which follows this rule is the 5th choice.
Answer choice 1 is incorrect because it contains three suns and three clouds.
Answer choice 2 is incorrect because it contains one sun and five clouds.
Answer choice 3 is incorrect because it contains two suns and six clouds.
Answer choice 4 is incorrect because it contains three suns and five clouds.

Therefore, the 5th choice is the correct answer.

74. Explanation
The correct answer is 2.

In this question, we can see that:


 All three figures consist of shapes divided into four parts.
 The symbols shared by each of the three figures are the smiley face and
the sun. These symbols are located in opposite places inside the figures.

The 2nd answer choice is the only one containing a smiley face and a sun located in
opposite places.

Therefore, the 2nd choice is the correct answer.

75. Explanation
The correct answer is 4.

In this question, we can see the following:


All three figures are shapes containing one diagonal inner line.
The inner line divides each shape into two unequal parts.
All three shapes contain a cloud, which is located above the inner line.

In the 4th answer choice, there is a shape with a diagonal inner line, dividing that
shape into two unequal parts. Moreover, this shape contains
a cloud located above the inner line.

The 1st and 2nd answer choices are incorrect since in both cases the inner line is not
diagonal and divides the shape into two equal parts.
The 3rd answer choice is incorrect as the cloud is located below the inner line while it
should be above the line.
The 5th answer choice is incorrect as it contains two inner lines instead of one.

Therefore, the 4th answer choice is the correct answer.

26
76. Explanation
The correct answer is 2.

In this question, we can see that:


 All three figures contain a shape with two inner lines that meet at a single
point.
 One of the inner lines is straight, while the other is curved.

The 1st answer choice is incorrect as the two inner lines are straight.
The 3rd answer choice is incorrect as this shape contains three inner lines.
The 4th answer choice is incorrect as the two inner lines inside the shape do not meet
at any point.
The 5th answer choice is incorrect as the two inner lines are curved.

In the 2nd answer choice, we can see a shape containing two inner lines that meet at
a single point. One of the lines is straight and the other is curved.

Therefore, the 2nd answer choice is the correct answer.

77. Explanation
The correct answer is 5.

In this question, we can see that all three figures in the top row consist of three
identical shapes that share a common area:

Similarly, in the 5th answer choice, we can see that all three circles share a common
area:

27
The rest of the answer choices do not contain an area that is shared by all three shapes.

Therefore, the 5th answer choice is the correct answer.

78. Explanation
The correct answer is 2.

In this question, we can see that:


 All three figures contain a shape with an inner arrow pointing to one of the
shape’s corners.
 In each of the figures, one of the shapes (either the inner arrow or the outer
shape) is white, while the other shape is dark blue.

In the 2nd answer choice, we can see a shape (triangle) with an inner arrow pointing
to one of its corners. In addition, the triangle is dark blue, while the arrow inside it
is white.

Therefore, the 2nd answer choice is the correct answer.

79. Explanation
The correct answer is 4.

In this question the three figures are all shapes that have perfect squares in the center.
The only answer choice that is like this is the 4th choice. The 2nd, 3rd and 5th choices
are incorrect as they contain inner shapes that are not square and have the wrong
number of sides. The 2nd choice is also incorrect as it contains a four sided shape that
is not a square but a diamond.

Therefore, the 4th choice is the correct answer.

80. Explanation
The correct answer is 4.

In this question all three figures are shapes with inverted corners (some of the shape's
corners point into the shape). The only answer choice that is like this is the 4th choice.
The rest of the choices only contain corners that point outward.

Therefore, the 4th choice is the correct answer.

28
81. Explanation
The correct answer is 1.

In this question the three figures all contain a shape with curved sides inside a shape
that only has straight sides. Notice that the inner and outer shapes are always different
colors. The only answer choice that is like this is the 1st choice. The 2nd choice is
incorrect because the inner shape is straight and outer shape is curved while the
opposite should be true. The 3rd choice is incorrect as both the inner and outer shapes
are the same color. The 4th choice is incorrect as both of the shapes have curved sides.
Finally, the 5th choice is incorrect as the both of the shapes have only straight sides.

Therefore, the 1st choice is the correct answer.

82. Explanation
The correct answer is 5.

In this question all three figures are shapes that have no inverted corners (all of the
shapes corners point outward). The only answer choice that is like this is the 5th
choice. The rest of the choices contain corners that point inverted corners (that
point into the shape).
Alternatively, another pattern this question presents is a series of shapes in which each
shape has one side less than the one before. The first shape is a green hexagon (6
sides), followed by a purple pentagon (5 sides) and an orange quadrilateral (3 sides).
The next shape in the series, therefore, would be a triangle (3 sides).
Therefore, the 5th choice is the correct answer.

83. Explanation
The correct answer is 2.

In this question the three figures are all purple octagons with a narrow plus sign in the
middle which is filled with various patterns. Notice that the octagon itself is always
solid purple. The only answer choice that is like this is the 2nd choice. The 1st choice
is incorrect as it has a plus sign in the center that is too large. The 3rd choice is
incorrect as it is a square and not an octagon. The 4th choice is incorrect because the
octagon itself is patterned while the plus sign isn't and it should be the opposite.
Finally, the 5th choice is incorrect as it has a wide border around the octagon shape
but shouldn't.

Therefore, the 2nd choice is the correct answer.

84. Explanation
The correct answer is 4.

In this question all three figures are orange circles each of which contains the

29
following three shapes: a yellow diamond, triangle and circle. The only answer choice
that is like this is the 4th choice. The 1st choice is incorrect as it contains two
diamonds and no triangle. The 2nd and 3rd choices are incorrect as they contain
trapezoids instead of triangles. Finally, the 5th choice is incorrect as it contains four
shapes instead of three (a trapezoid in addition to all of the other shapes).

Therefore, the 4th choice is the correct answer.

85. Explanation
The correct answer is 2.

In this question all three figures are triangles each of which contains the following
three shapes: a tilted square, a triangle and a circle. The only answer choice that is like
this is the 2nd choice. The 1st, 3rd and 4th choices are incorrect as they contain
diamonds instead of squares, and the 5th choice is incorrect as it contains two
triangles but no square.

Therefore, the 2nd choice is the correct answer.

86. Explanation
The correct answer is 2.

The three figures in this question include a bigger shape in the middle with smaller
shapes on top and at the bottom. The smaller shapes are a smaller version of the
upper half of the bigger shape.

This eliminates answer choice 1, as the smaller shapes are different from each other
and from the bigger shape.
Answer choice 5 is also eliminated, as the smaller shapes (pentagons) are the same as
the entire bigger shape (a pentagon), not the upper half.
Answer choice 3 is also wrong, as the smaller shapes above and below the bigger
shape show the entire bigger shape, only rotated.

We are now left with answer choices 2 and 4.


If you look closely, you will see that in answer choice 4, the smaller shape at the
bottom is turned upside down, as if it is the lower half of the bigger shape. Since it
must be the upper half, this is incorrect.

The 2nd choice is the only one left that follows these rules, so it is the correct answer.

87. Explanation
The correct answer is 3.

30
All three figures in this question contain rectangles that are divided into four, with a
circle and a rhombus inside.

This eliminates answer choices 2, 4, and 5, as they contain shapes that are not a circle
and rhombus.

How do we choose between answer choices 1 and 3?


Look again at the three figures in the top row - what else do they have in common?
We can see that in all three figures, the circle and the rhombus appear next to each
other.
The only answer choice which follows this rule is the 3rd choice.
Therefore, the 3rd choice is the correct answer.

88. Explanation

The correct answer is 5.

All three figures in this question contain shapes divided (approximately) into
one-third (1/3) and two-thirds (2/3) parts.
The only answer choice which follows this rule is the 5th choice.
The 1st, 2nd, and 3rd choices are all divided evenly into halves, so they are incorrect.
The 4th choice is clearly not divided in the same way as the shapes in the question, as
the smaller part is much smaller and the larger part is much larger.
Therefore, the 5th choice is the correct answer.

89. Explanation
The correct answer is 1.

In this question, we can see that:


• All three figures include a circle, a square and a rhombus
• The shapes are either white, dark or striped
• Each shape and shading appear only once in each figure

In answer choice 2 we have a pentagon instead of a circle, and in answer choice 3 we


have a heart instead of a rhombus, so they can be eliminated.
In answer choices 4 and 5 either the shading or the shape repeat, so they can
be eliminated as well.

Therefore, the 1st choice is the correct answer.

90. Explanation
The correct answer is 4.

In this question all three figures have:

31
• A rectangle at the bottom,
• A circle in the middle, and –
• A triangle at the top.
The only answer choice which follows these rules is the 4th choice.

Therefore, the 4th choice is the correct answer.

91. Explanation
The correct answer is 5.

Each of the three figures in this question has two corners. A corner is where two
straight lines meet.

In answer choice 1 the shape has three such corners.

The shape in answer choice 2 has five such corners.

Answer choice 3 has no corners at which two straight lines meet.


Answer choice 4 has three such corners.

Therefore, the 5th choice is the correct answer.

32
92. Explanation
The correct answer is 5.

Each of the figures in this question has six sides and an oval shape on the inside.

The only answer choice which follows these rules is the 5th choice.

All of the other answer choices contain shapes with fewer than six sides, and the
shapes inside them are not oval.

Therefore, the 5th choice is the correct answer.

93. Explanation
The correct answer is 3.

All three figures in this question contain:


• An outer shape which is a square
• An inner shape whose top part is textured, and
• A background whose bottom part is textured

The only answer choice which follows all of these rules is the third choice.

The first choice is incorrect because the textured parts are the bottom of the inner
shape and the top of the background instead of the other way around.
The second choice is incorrect because the outer shape is a circle instead of a square.
The fourth choice is incorrect because both parts of the inner shape are textured, and
the background has no textured part at all.
The fifth choice is incorrect because there is no inner shape, and the bottom part of
the background has 2 different textures. (an alternative explanation is that the inner
shape is a rectangle and it is fully textured, instead of just the top half.)
Therefore, the 3rd choice is the correct answer.

94. Explanation
The correct answer is 2.

All three figures in this question contain:


• An outer shape which is a circle,
• A 5-sided shape (pentagon) inside the circle, and –
• A smaller 4-sided shape (quadrilateral) inside the pentagon

The only answer choice which follows these rules is the 2nd choice.
Therefore, the 2nd choice is the correct answer.

33
95. Explanation
The correct answer is 5.

In this question all three figures are shapes with three squares, some appearing in 3D,
with one shape in each square.
The squares in all three figures contain a sun, a heart and a banner.

Answer choice 1 is a shaped like a can and does not contain three squares.
Answer choice 2 does contain three squares, but the banner does not look like the one
in the figures above.
Answer choice 3 does not contain three squares with a shape in each one.
Answer choice 4 contains an octagon and not three squares.

Therefore, the 5th choice is the correct answer.

96. Explanation
The correct answer is 5.

In this question, we can see that:


All three figures are comprised of two shapes.
One of the shapes is located in the front and the other shape in the back.
The shape in the back has one side more than the shape in the front.

We can see that in the 5th answer choice, the shape in the back has six sides, while
the shape in the front has five sides. This means that the shape in the back has one
side more than the shape in the front.

Therefore, the 5th answer choice is the correct answer.

97. Explanation
The correct answer is 1.

In this question, we can see that:


 The three figures are squares divided into nine smaller parts.
 In each figure, there are three Xs and one circle.
 In each figure, the three Xs are organized in a sequence of a straight or a
diagonal line.
 In each figure, the circle is located in one of the bigger square’s corners.
In the 1st answer choice, we can see a square divided into nine smaller parts. This
square contains three Xs and one circle. We can see that the Xs are organized in
a sequence of a straight line and that the circle is located in one of the bigger
square’s corners.
Therefore, the 1st answer choice is the correct answer.

34
98. Explanation
The correct answer is 3.

In this question, you can see that:

•Each of the three figures consists of two similar shapes, one on top of the other.
•The top shape is similar to the bottom shape, only rotated 90 degrees clockwise.

In the 3rd answer choice, you can see two similar shapes (trapezoids), one on top of
the other. In addition, the top trapezoid is similar to the bottom one, only rotated 90
degrees clockwise.

Therefore, the 3rd answer choice is the correct answer.

The first answer choice shows two shapes side by side instead of one on top of the
other, and therefore it cannot be correct.

The second answer choice has one shape on top, but it has only been rotated through
45 degrees instead of 90 degrees, so it cannot be correct.

The fourth answer choice shows two shapes, where the top has been rotated through
180 degrees instead of 90 degrees, so it cannot be correct.

In the fifth answer choice, the top shape has been rotated 90 degrees anticlockwise so
the top arrow is pointing west instead of east. Therefore, it cannot be correct.

99. Explanation
The correct answer is 5.

In this question, we can see that:


 All three figures are shapes divided into smaller parts.
 In each shape, half of the smaller parts are colored: In the circle and the
diamond, four out of eight parts are colored. In the rectangle, two out of
four parts are colored.

In the 5th answer choice, we can see a hexagon divided into six smaller
parts. Three out of six parts are colored, which means that half of the hexagon’s total
area is colored.

Therefore, the 5th answer choice is the correct answer.

100. Explanation
The correct answer is 3.
All three figures in this question are divided into two equal halves. In addition, the

35
separating line is a diagonal line slanted up to the left.
The third figure is divided into two equal halves, and the separating line is a diagonal
line which slants up to the left. Thus, this is the correct answer.
The first answer choice is incorrect as the separating line slants up to the right.
The second answer choice is incorrect as the separating line is horizontal when it
should be diagonal.
The fourth answer choice is incorrect as the separating line slants up to the right.
The fifth answer choice is incorrect as the figure is not divided into two equal halves.

101. Explanation
The correct answer is 1.

All three figures in this question are comprised of:

 An outer shape made up of curved lines only.


 Two identical black inner shapes made up of straight lines only.

The only answer choice which follows these rules is the 1st one. The outer shape is
made up of curved lines only, and the two black inner shapes (hexagons) are made
up of straight lines only.

Therefore, the 1st choice is the correct answer.

102. Explanation
The correct answer is 2.

In this question, all three figures above consist of a big shape in the middle, with
the same two smaller shapes, one on the right and one on the left. In addition, notice
that all the figures are aligned, and that the smaller shapes are in the same direction as
the big shape.

The second figure consists of a big pentagon in the middle with two smaller
pentagons on its right and left. Notice that the small pentagons are in the same
direction as the big pentagon, and that the whole figure is aligned. Thus, this is the
correct answer.

The first answer choice is incorrect as the two smaller rectangles are located above
and beneath the big rectangle, instead of being located on its sides.

The third answer choice is incorrect as the small pentagons are pointing downwards
while the big pentagon is pointing upwards, when instead they all should be on the
same direction.

The fourth answer choice is incorrect as the figure is not aligned but rather slants to

36
the right.

The fifth answer choice is incorrect as the smaller shapes (rectangles) are not the same
as the big shape (the pentagon).

103. Explanation
The correct answer is 1.

In this question, you can see that the three figures above consist of three shapes each,
which are set in a diagonal manner: a circle, a trapezoid, and a star. In addition, in
each figure, the shapes (the circle, the trapezoid, and the star) are either dark
blue, white, or checkered. Notice that there is no match between a specific shape and
a specific color, and that no two shapes in the same figure have the same filling.

The first figure consists of the three correct shapes: the circle, the trapezoid, and the
star. These shapes are correctly set diagonally. In addition, these shapes have the
correct colors: white (the trapezoid), dark blue (the circle), and checkered (the star).
Thus, this is the correct answer.

The second and third answer choices are incorrect as although they consist of the
correct shapes and have the correct colors, the shapes in them are not set diagonally,
and therefore they can be eliminated.

The fourth and fifth answer choices are incorrect as although they have the correct
shapes which are correctly set diagonally, they do not have the correct colors: the
fourth answer choice is not white, and the fifth answer choice is not checkered.
Therefore, these answer choices can be eliminated.

104. Explanation
The correct answer is 3.

In this question, the three figures above have straight sides and are filled in
with diagonal lines that slant up to the right.

The third answer choice follows these conditions: it is a shape that consists of only
straight sides, and is filled in with diagonal lines slanted up to the right. Therefore,
this is the correct answer.

The first, fourth, and fifth answer choices are incorrect as they all consist of curved
lines as well as straight lines.
The second answer choice is incorrect as although it consists of straight lines only, it
is filled in with diagonal lines slanted up to the left instead of being slanted up to the
right.

37
Tip: If you find it difficult to infer the rule connecting the three figures above, it
might be useful to look at the answer choices to see if there are any unusual figures. In
this question, you can see that only answer choices 2 and 3 have straight lines, so you
can check if this is the correct rule.

105. Explanation
The correct answer is 4.

In this question, the three figures above all contain a diagonal line that connects one
of the shape’s corners with one of the shape’s sides. Notice that the line does
not connect two corners.

The shape in the fourth answer choice contains a diagonal line that stretches from its
bottom left corner to the middle of its upper right side. Therefore, this is the correct
answer.

The first answer choice is incorrect as the line in this shape connects two sides and
does not reach a corner.
The second answer choice is incorrect as the line in this shape connects only corners
and does not reach one of the shape’s sides.
The third answer choice is incorrect as there are two lines in this shape while there
should be only one.
The fifth answer choice is incorrect as the line in this shape connects two sides and
does not reach a corner.

106. Explanation
The correct answer is 3.

In this question, the three figures above all contain two inner lines
that intersect inside the shape. Notice that the inner lines divide each figure
into four parts (not necessarily equal).

The third answer choice is a pentagon which contains two inner lines that intersect
inside it. In addition, these lines divide the pentagon into four parts. Therefore, this is
the correct answer.

The first and the second answer choices are incorrect as the two inner lines in these
figures do not intersect.
The fourth answer choice is incorrect as this figure contains three inner lines instead
of two.
The fifth answer choice is incorrect as although this figure is divided into four parts, it
contains three inner lines instead of two.

38
107. Explanation
The correct answer is 1.

In this question, the three figures above are divided into two parts: a white part and
a checkered part. In addition, within the white part, there is a small white
shape which is the same as the outer figure.

The first answer choice is a pentagon divided into a white part and a checkered part.
Within the white part there is a small pentagon, which is the same as the outer figure.
Therefore, this is the correct answer.

The second answer choice is incorrect as the small shape is the same as the shape of
the white part (a trapezoid), while it should be the same as the outer figure (a
hexagon).
The third answer choice is incorrect as the small shape is located within the checkered
part instead of the white part.
The fourth answer choice is incorrect as the small shape (a triangle) is checkered
instead of being white. In addition, the outer figure (except for the inner small triangle)
is white, while it should be both white and checkered.
The fifth answer choice is incorrect as the small shape (a triangle) is different than the
outer figure (a parallelogram).

108. Explanation
The correct answer is 5.

In this question, we can see that:


 The three figures are circles containing two inner shapes, one to each side of
an inner line that cuts the circles in half.
 One of the inner shapes is white, while the other inner shape is black.
 In each figure, the two inner shapes have a different number of sides.

The 1st and 3rd answer choices are incorrect as the two inner shapes in these choices
have the same number of sides.
The 2nd and 4th answer choices are incorrect as the two inner shapes in these choices
are the same color.
The 5th answer choice is a circle containing one black shape and one white shape. In
addition, these two shapes have a different number of sides (pentagon with five
sides and triangle with three sides).

Therefore, the 5th answer choice is the correct answer.

109. Explanation
The correct answer is 4.
In this question, all pictures contain a square with a truncated top-right corner.

39
The only choice that matches this description is 4, and therefore, it is the correct
answer.
o Answer 2 has a truncated top-right corner, but it is not a square since its
top-left corner is rounded.
o Answer 5 also has a truncated top-right corner, but it also has a truncated
bottom-left corner.

110. Explanation
The correct answer is 3.

In this question you can see that:

-Each of the above three figures has an outer shape with straight lines only.
-Each of the figures contain two inner circles.
-In each figure, the two inner circles have the same filling.

Answer choice 3 is the correct answer as its outer shape is comprised of straight lines
only, and as it contains two inner circles with the same filling.

Answer choices 1 and 4 are incorrect as they are comprised also of curved lines.
Answer choice 2 is incorrect as its inner circles do not have the same filling.
Answer choice 5 is incorrect as it contains three inner circles instead of two.

111. Explanation
The correct answer is 4.
All the figures follow these basic rules:
 All the figures in the left quadrant are white.
 All the figures in the upper quadrant are black.
 All the figures in the right quadrant are orange.
 All the figures in the lower quadrant are blue.
The only figure that applies these rules is image 4 - the correct answer.

112. Explanation
The correct answer is 4.
In this question, all pictures contain figures with four lines, two of which are straight
lines, and the other two are curved.
The only figure with these features among the answer choices is answer 4, and
therefore, it is the correct answer.

113. Explanation
The correct answer is 3.
In this question, the pattern is as follows:

40
 Each picture presents a big figure and a smaller cut figure linked to the top of
it.
 The small cut figure is identical to the bottom part of the bigger figure, and it
is upside down.
The only choice that matches this pattern is the 3rd one, and therefore, it is the correct
answer.
o Answer 4 is incorrect because even though it seems to follow the same pattern,
the cut smaller cut figure is not upside down as required.

114. Explanation
The correct answer is 2.
In this question, the pattern is as follows:
 Each picture consists of one big figure, one smaller figure inside of it, and one
even smaller figure inside of that.
 The number of sides of the smallest internal figure is the number exactly
halfway between the number of sides of the middle figure and the big, external
figure.
 For example, in the right picture, the big, external figure has three sides, and
the middle figure has seven sides, so the smallest internal figure has five sides
(five is exactly halfway between three and seven).
The 2nd choice is the correct answer because it is the only option that follows the
pattern, as its external, big figure has five sides, its middle figure has seven sides, and
accordingly, its internal, smallest figure has six sides, which is the number halfway
between the two.

115. Explanation
The correct answer is 4.
In this question, the pattern is as follows:
 Each picture contains one small figure inside a bigger figure.
 The number of sides of the internal figure is exactly half the number of sides
of the external figure.
 For example, in the left picture, the external figure has six sides, and therefore
the internal figure has three sides (a triangle).
The correct answer is 4 because it is the only one that follows the pattern - the big
external star has ten sides, and the small internal figure has five sides (a pentagon).

116. Explanation
The correct answer is 3.
In this question, the features that repeat for all three pictures are:
 All pictures consist of one blank isosceles triangle, one blank square, and one
or two small black hearts.
 The bottom-left corner of the square and the top corner of the triangle always
contains a black heart.

41
Answer 3 is the only choice that has all these features, and therefore, it is the correct
answer.
o Answer 2 is incorrect because the triangle covers the square.
o Answer 4 is incorrect because the square’s corners are rounded (meaning that
it is not a square).

117. Explanation
The correct answer is 2.

The three figures in this question include a bigger shape in the middle with smaller
shapes on top and at the bottom. The smaller shapes are a smaller version of the
upper half of the bigger shape.

This eliminates answer choice 1, as the smaller shapes are different from each other
and from the bigger shape.
Answer choice 5 is also eliminated, as the smaller shapes (pentagons) are the same as
the entire bigger shape (a pentagon), not the upper half.
Answer choice 3 is also wrong, as the smaller shapes above and below the bigger
shape show the entire bigger shape, only rotated.

We are now left with answer choices 2 and 4.


If you look closely, you will see that in answer choice 4, the smaller shape at the
bottom is turned upside down, as if it is the lower half of the bigger shape. Since it
must be the upper half, this is incorrect.

The 2nd choice is the only one left that follows these rules, so it is the correct answer.

118. Explanation
The correct answer is 5.
In this question, all pictures feature the following pattern:
 All pictures contain five identical figures in the same formation: they are all
arranged with one shape in the middle, one directly above it, one directly
below it, one directly to the left, and one directly to the right. Answer 3 can
therefore be eliminated as its formation is different.
 All figures are quadrilaterals (shapes with 4 straight sides). Answers 1 and 4
can also be eliminated at this point as the figures in them are not
quadrilaterals.
 The middle square in all pictures is black, and the rest of the squares are either
grey or white (more specifically – 2 grey and 2 white). Answer 2 can be
eliminated since its middle square is grey, and 2 of the other squares are black
instead of the other way around.
We are left with the 5th choice as the only option that matches the pattern, and
therefore, it is the correct answer.

42
119. Explanation
The correct answer is 4.
In this question, the pattern is as follows:
 Each picture contains one big figure and another smaller identical figure inside
it.
 Each small inner figure contains one vertical line and two diagonal lines, all
intersecting at a single point. Notice that these lines are only inside the inner
figures.
o Answer 1 is incorrect since there is a horizontal line instead of a vertical line,
and there is more than one intersection point.
o Answer 2 is incorrect since the two diagonal lines are stretching from the
corners of the external figure and not only placed inside the smaller figure.
o Answer 3 is incorrect since the internal figure is different from the external
one.
o Answer 5 is incorrect since there are no diagonal lines.
We are left with the 4th choice as the only answer that matches the pattern, and
therefore, it is the correct answer.

120. Explanation
The correct answer is 5.
 All pictures consist of a square divided into small triangles (18 triangles to be
precise).
 In each picture, exactly five of the small triangles forming the square are black,
whilst the rest of them are white.
 Notice that some of the black triangles, when close to one another, form larger
shapes.
The only option that matches this description is the 5th choice because its picture has
two black squares on the corners, each comprised of two black triangles and another
black triangle in the middle. Therefore, this is the correct answer.
o Answer 2 is incorrect because it has five white triangles, and the rest are black
instead of the other way around.
o Answer 4 is incorrect because the square in it is comprised of 12 larger
triangles in different orientations.

121. Explanation
The correct answer is 4.
In this question, each picture presents four arrows, and the pattern is as follows:
 In all pictures, each arrow points in a different direction: one up, one down,
one left, and one right. Answer 5 can therefore be eliminated as two arrows
point to the right.
 In each picture, one of the arrows is brighter than the others. Answer 1 can
therefore be eliminated as two of the arrows are brighter.

43
 The brighter arrow in all pictures is pointing to the side, left or right, and not
up or down. Answer 3 can be eliminated as the brighter arrow is pointing
down.
In all pictures, the order of the arrows, from left to right, remains the same, only
shifted Down, Up, Left, Right:


It is enough to only notice that the two arrows pointing up and down are next
to each other in all pictures. Answer 2 can now be eliminated as these arrows
are apart and the order is different.
We are left with the 4th choice, which is the correct answer.

122. Explanation
The correct answer is 1.
In this question, each picture contains a large figure with two smaller versions of the
same figure, one indented into it and the other protruding out of it. The figure appears
as one shape with one continuous line.
The only option that matches this pattern is the 1st choice since it consists of a big
hexagon with two smaller hexagons, one indented into it and the other protruding
from it.
o Answer 2 is incorrect because the indented figure is not the same shape as the
big figure, and the protruding one also has shorter sides proportionally.
o Answer 3 is incorrect because the smaller shapes are not the same shape as the
large figure.
o Answer 4 is incorrect because the figure does not consist of one continuous
line; the internal shape is enclosed in the large figure rather than being
indented.

44
o Answer 5 is incorrect because both smaller shapes are protruding and one of
them needs to be indented.

123. Explanation
The correct answer is 4.
In this question, the pattern is as follows:
 Each picture contains a white figure and a small black square within the edge
of the bigger figure.
 The bigger figure is comprised of at least one curved line.
o Answers 1 and 2 are incorrect because the big, white figures do not contain a
rounded line.
o Answer 3 is incorrect because the black square is partially hidden by the big
white figure; it is not a full square, and it is behind the white figure.
o Answer 5 is incorrect because the black square is not on the edge of the big
white figure. Also, the figure contains additional lines dividing the big figure
into parts.
We are left with the 4th choice, which is the correct answer.

124. Explanation
The correct answer is 2.
In this question, all pictures contain a big octagon, divided into four black triangles,
four white rectangles, and a white square in the middle. The pictures follow this
pattern:
 The middle square of each picture contains a black triangle
 One of the rectangles contains a quadrilateral, and another one contains a
pentagon.

o Answer 1 is incorrect because it contains a hexagon instead of a pentagon.


o Answer 3 is incorrect because the rotation of the octagon is different (or that
the lines dividing the octagon are different).
o Answer 4 is incorrect because the triangle in the middle square is white.
o Answer 5 is incorrect because the middle square contains a quadrilateral
instead of a triangle.
We are left with the 2nd choice, which is the correct answer.

125. Explanation
The correct answer is 3.
In this question, the pattern is as follows:
 Each picture contains two small figures inside one bigger figure.
 The number of sides of the bigger figure is equal to the sum of the two smaller
figures’ number of sides.
 For example, the left picture consists of two three-sided figures (triangles)
inside of a big six-sided figure.

45
The correct answer is 3 because it is the only choice that follows the pattern (the big
arrow consists of seven sides, and the small triangle and square consist of a sum of
seven sides as well).

126. Explanation
The correct answer is 3.
In this question, the pattern is as follows:
 Each picture presents a figure and a varying number of small black squares
under it.
 The black squares are located in accordance with the ends of each line of the
above figure (for closed figures, it is therefore in its corners).
Since this pattern may be a bit tricky and not certain, the best solving method would
be the elimination of answer choices:
o Answer 1 is incorrect because there are small black triangles instead of
squares.
o Answer 2 is incorrect because the small black squares’ orientation is different.
o Answer 4 is incorrect because the black shape is not a square, and it is too
wide (and therefore different).
o Answer 5 is incorrect because there are fewer small black squares than corners
and lines of the figure, and the middle black square is not in accordance with
any part of the figure.
We are left with the 3rd choice, which is the correct answer because the two black
squares under the line are in accordance with the two ends of the line.

127. Explanation
The correct answer is 5.
All pictures consist of a black figure and another identical figure rotated by 180° next
to it.
o Answer 1 is incorrect because it contains two identical figures that are
mirroring each other; if the second figure were rotated 180°, the figures would
have the same orientation instead of mirroring each other.
o Answer 2 is incorrect because it also contains two identical figures mirroring
one another instead of a 180° rotation.
o Answer 3 is incorrect because the two figures are not identical.
o Answer 4 is incorrect because the second figure is not a 180° rotation of the
first.
We are left with the 5th choice, which, even though the two figures have the exact
same orientation, this figure remains that way after a 180° rotation, and therefore, it is
the correct answer.

128. Explanation
The correct answer is 1.
In this question, all pictures consist of a circle divided with diagonal lines into 4 parts,
with the following features:

46
 Each picture contains one circle, one quadrilateral, one triangle, and one
arrow.
 The figures are coloured black, white, or grey, and each picture contains at
least one of each colour.
 In each picture, the shape that is being pointed at by the arrow is white.
The only answer that matches the pattern is the 1st choice, and therefore, it is the
correct answer.
o Answer 2 is incorrect since the shape the arrow points to is black instead of
white.
o Answer 3 also follows all pattern features but the lines dividing the circle are
not diagonal, as in the given pictures.

129. Explanation
The correct answer is 3.
In this question, the pattern is that each of the pictures contain the exact same figure,

in its exact size and orientation. The repeated figure is:


That means that the same figure is repeated in each picture, so the answer should also
contain this exact figure.

The only answer containing the repeated figure is the 3rd choice, and therefore, it is
the correct answer.

130. Explanation
The correct answer is 1.
In this question, the pattern is as follows:
 Each picture contains one big, white figure and a varying number of smaller
grey figures inside of it.

47
 If you add up the total number of sides of all the internal shapes, it must be
double the number of sides of the big, white figure that they are inside of.
 For example, the left picture consists of a big, white square (four sides), one
triangle (three sides), and one pentagon (five sides) within it. Therefore, the
total number of sides of the internal figures is eight (5 + 3 = 8), which is two
times the external number of sides (four).
The correct answer is 1 because it is the only choice that follows the pattern; the big
figure has three sides, and the internal figure has six sides.

131. Explanation
The correct answer is 1.

All three figures in this question are comprised of:

 An outer shape made up of curved lines only.


 Two identical black inner shapes made up of straight lines only.

The only answer choice which follows these rules is the 1st one. The outer shape is
made up of curved lines only, and the two black inner shapes (hexagons) are made
up of straight lines only.

Therefore, the 1st choice is the correct answer.

132. Explanation
The correct answer is 5.

Each of the three figures in this question has two corners. A corner is where two
straight lines meet.

In answer choice 1 the shape has three such corners.

The shape in answer choice 2 has five such corners.

48
Answer choice 3 has no corners at which two straight lines meet.
Answer choice 4 has three such corners.

Therefore, the 5th choice is the correct answer.

133. Explanation
The correct answer is 5.

Each of the figures in this question has six sides and an oval shape on the inside.

The only answer choice which follows these rules is the 5th choice.

All of the other answer choices contain shapes with fewer than six sides, and the
shapes inside them are not oval.

Therefore, the 5th choice is the correct answer.

49
134. Explanation
The correct answer is 2.
In this question, all pictures consist of small black and white cubes forming a
structure.
The common features for all three pictures are as follows:
 All pictures consist of exactly five squares.
 All pictures contain two white and three black squares.
 Each black square shares at least one of its sides with another black square
whilst the white squares are separated.
o Answer 5 can be eliminated as it contains six squares, four of which are black.
o Answers 1 and 3 can now be eliminated as they consist of three white and two
black squares (instead of the other way around).
o Answer 4 can be eliminated since the white squares share a side and one of the
black squares shares none of its sides with another black square.
We are left with the 2nd choice, which is the correct answer.

135. Explanation
The correct answer is 5.
Each picture includes two shapes; an outer shape patterned with diagonal lines, and
another inner shape.
In addition, the inner shape has one side less than the outer shape.
Answer 5 fits the pattern and therefore it is the correct answer.
Answers 1 and 4 are wrong because the backgrounds are white rather than the inner
shape.
Answer 2 is wrong because the inner shape has the same number of edges as the
shape on the outside.
Answer 3 is wrong because the outer shape (the triangle) has one less edge than the
inner shape (the parallelogram), instead of the other way around.

136. Explanation
The correct answer is 3.
All pictures consist of two identical figures mirroring one another horizontally. The
only answer that follows that pattern is the 3rd choice, and it is, therefore, the correct
answer.
o Answer 1 contains two identical figures with the same orientation (no
mirroring).
o In answer 2, the figures do not mirror one another – just an upside-down copy
of one another.
o In answer 4, the figures are different in size.
o In answer 5, the figures touch each other and share a side – in contrast to all
three pictures in the question.

50
137. Explanation
The correct answer is 5.
In this question, the pattern is that each of the pictures contains the exact same figure,

in its exact size and orientation. The repeated figure is this:


That means that the same figure is repeated in each picture, so the answer should also
contain this exact figure.

The only option containing the repeated figure is the 5th choice, and therefore, it is
the correct answer.

138. Explanation
The correct answer is 5.
In this question, the pattern is as follows:
 Each picture presents a rectangle with a vertical dashed line in its middle,
dividing it into two parts. Answers 2 and 4 can therefore be eliminated at this
point as they do not follow this rule.
 The right part of each rectangle is divided into twice as many parts as the left
part of that rectangle. For example, if the left part is divided into 4 parts, the
right part is divided into 8 parts (like both the left and the middle pictures).
Answer 5 is the only option that follows the pattern, as the left side of the rectangle is
divided into three parts, and the right side is divided into six parts. Therefore, it is the
correct answer.

139. Explanation
The correct answer is 4.
In this question, all pictures feature the following pattern:
 All pictures contain one long figure and three shorter different figures. At this
point, answers 1 and 5 can be eliminated as they contain more or less than
three short figures. Answer 3 can also be eliminated since the three short
figures are the same as the longer figure.
 The three shorter figures are in front of the long figure, hiding part of it.
Answer 2 can therefore be eliminated as the three short figures are behind the
long figure, which hides them instead of the other way around.

51
We are left with the 4th choice, which is the correct answer.

140. Explanation
The correct answer is 5.
In this question, all pictures consist of a circle divided by horizontal and vertical lines
into 4 parts, with the following features:
 All pictures contain two triangles and one square.
 The two triangles are of the same colour, and the square is of a different colour
than the triangles. Answer 1 can therefore be eliminated now as the triangles
are in different colours.
 The triangles are located diagonally across from one another. Answer 2 can
therefore be eliminated at this point as the triangles are next to each other.
 The orientation of the two triangles is different from one another. Thus,
answer 3 can be eliminated, as both triangles have the same orientation.
o Although answer 4 matches all of these pattern features, the lines dividing the
square are diagonal, unlike those of the given three pictures.
We are left with the 5th choice, which is the correct answer and the only picture that
follows the pattern.

141. Explanation
The correct answer is 2.
In this question, each picture presents a black dot, and several twisted lines come out
of it. The pattern is as follows:
 The number of twisted lines coming out of the dot is equal to the number of
vertices of each twisted line.
 Another interpretation is that the number of twisted lines coming out of the dot
is one less than the number of straight lines forming each twisted line.
 For example, in the left pictures, four twisted lines are coming out of the dot,
and each of them is formed out of five straight lines and contain four vertices.
The only option that matches the pattern is the 2nd choice, as there is one twisted line
coming out of the dot, and it is formed out of two straight lines and contains one
vertex.

142. Explanation
The correct answer is 3.
In this question, the pattern is as follows:
 All pictures contain two big white figures with one shared side.
 Inside each figure, there is a small black figure, identical to the other figure in
that picture.
 Each small black figure is mirroring its bigger identical figure (its orientation
is opposite).
The only answer that matches the pattern is the 3rd choice, and therefore, it is the
correct answer.

52
o Answer 1 is incorrect since each small black figure is inside its identical big
figure instead of the other way around.
o Answer 4 is incorrect since the small black figures are not mirroring their
identical big figures (their orientation is the same as the big figures).

143. Explanation
The correct answer is 5.

In this question, we can see that:


 The three figures are circles containing two inner shapes, one to each side of
an inner line that cuts the circles in half.
 One of the inner shapes is white, while the other inner shape is black.
 In each figure, the two inner shapes have a different number of sides.

The 1st and 3rd answer choices are incorrect as the two inner shapes in these choices
have the same number of sides.
The 2nd and 4th answer choices are incorrect as the two inner shapes in these choices
are the same color.
The 5th answer choice is a circle containing one black shape and one white shape. In
addition, these two shapes have a different number of sides (pentagon with five
sides and triangle with three sides).

Therefore, the 5th answer choice is the correct answer.

144. Explanation
The correct answer is 2.

In this question, all three figures above consist of a big shape in the middle, with
the same two smaller shapes, one on the right and one on the left. In addition, notice
that all the figures are aligned, and that the smaller shapes are in the same direction as
the big shape.

The second figure consists of a big pentagon in the middle with two smaller
pentagons on its right and left. Notice that the small pentagons are in the same
direction as the big pentagon, and that the whole figure is aligned. Thus, this is the
correct answer.

The first answer choice is incorrect as the two smaller rectangles are located above
and beneath the big rectangle, instead of being located on its sides.

The third answer choice is incorrect as the small pentagons are pointing downwards
while the big pentagon is pointing upwards, when instead they all should be on the
same direction.

53
The fourth answer choice is incorrect as the figure is not aligned but rather slants to
the right.
The fifth answer choice is incorrect as the smaller shapes (rectangles) are not the same
as the big shape (the pentagon).

145. Explanation
The correct answer is 1.

In this question, the three figures above are divided into two parts: a white part and
a checkered part. In addition, within the white part, there is a small white
shape which is the same as the outer figure.

The first answer choice is a pentagon divided into a white part and a checkered part.
Within the white part there is a small pentagon, which is the same as the outer figure.
Therefore, this is the correct answer.

The second answer choice is incorrect as the small shape is the same as the shape of
the white part (a trapezoid), while it should be the same as the outer figure (a
hexagon).
The third answer choice is incorrect as the small shape is located within the checkered
part instead of the white part.
The fourth answer choice is incorrect as the small shape (a triangle) is checkered
instead of being white. In addition, the outer figure (except for the inner small triangle)
is white, while it should be both white and checkered.
The fifth answer choice is incorrect as the small shape (a triangle) is different than the
outer figure (a parallelogram).

146. Explanation
The correct answer is 1.

In this question, we can see that:


 The three figures are squares divided into nine smaller parts.
 In each figure, there are three Xs and one circle.
 In each figure, the three Xs are organized in a sequence of a straight or a
diagonal line.
 In each figure, the circle is located in one of the bigger square’s corners.

In the 1st answer choice, we can see a square divided into nine smaller parts. This
square contains three Xs and one circle. We can see that the Xs are organized in
a sequence of a straight line and that the circle is located in one of the bigger
square’s corners.

Therefore, the 1st answer choice is the correct answer.

54
147. Explanation
The correct answer is 2.

In this question, we can see that:


 All three figures consist of shapes divided into four parts.
 The symbols shared by each of the three figures are the smiley face and
the sun. These symbols are located in opposite places inside the figures.

The 2nd answer choice is the only one containing a smiley face and a sun located in
opposite places.

Therefore, the 2nd choice is the correct answer.

148. Explanation
The correct answer is 2.
 In this question, all pictures present a square, with a possibility of shifting
some of its corners inwards.
 The basic pattern that repeats for all pictures is that the length of the line
forming the figure is the same.
 Notice that even though the shape of each figure is different, the only
difference is the inward shift of the lines at the corners. Therefore, the length
of the line that forms the perimeter is equal for each shape.
The only answer that matches this description is the 2nd choice, as it is also a
would-be square with a corner indented, resulting in the same length in line that forms
the perimeter.
o Answers 1, 3, and 4 are incorrect because they are not formed from one
continuous line.
o Answer 5 is incorrect because it is not a square, and the line that forms the
perimeter is, therefore, shorter in total length.

149. Explanation
The correct answer is 5.
Each of the given pictures is made by using exactly 3 lines.
Answer 5 is the only answer made using 3 lines, and therefore it is the correct answer.

150. Explanation
The correct answer is 3.

In this question, you can see that:

•Each of the three figures consists of two similar shapes, one on top of the other.
•The top shape is similar to the bottom shape, only rotated 90 degrees clockwise.

In the 3rd answer choice, you can see two similar shapes (trapezoids), one on top of

55
the other. In addition, the top trapezoid is similar to the bottom one, only rotated 90
degrees clockwise.

Therefore, the 3rd answer choice is the correct answer.

The first answer choice shows two shapes side by side instead of one on top of the
other, and therefore it cannot be correct.

The second answer choice has one shape on top, but it has only been rotated through
45 degrees instead of 90 degrees, so it cannot be correct.

The fourth answer choice shows two shapes, where the top has been rotated through
180 degrees instead of 90 degrees, so it cannot be correct.

In the fifth answer choice, the top shape has been rotated 90 degrees anticlockwise so
the top arrow is pointing west instead of east. Therefore, it cannot be correct.

151. Explanation
The correct answer is 5.

In this question you can see that:


- All figures are circles that have lines on both sides.
- All the figures have the same number of lines on each side.

Answer choice 5 has four lines on both the right side and on the left side of the circle,
and it is therefore the correct answer.

Answer choices 1, 2 and 3 are immediately eliminated since they do not have the
same number of lines on the left and right side of the circle.
After eliminating these answers you are left with answer choices 4 and 5, which have
the same number of lines. Since it seems that answer choice 4 could also be correct
because it has the same number of lines on each side of the circle (zero lines), you
must find another rule in order to eliminate one answer. As all the top figures have
lines, the correct answer must have lines and therefore answer 4 is eliminated.

152. Explanation
The correct answer is 4.
In this question, all pictures can be comprised of a big square and two isosceles
triangles that are exact quarters of the big square, such as this one:

56
*Note that even though the left figure has four such triangles in it, it can be comprised
of only two, placed one against the other, thus creating the other two with the square’s
sides: illustration.
The only option that matches that pattern is the 4th choice, and therefore, it is the
correct answer.
o Answers 1 and 3 are incorrect because they are comprised of only one such
triangle.
o Answer 2 is incorrect because it is comprised of three such triangles.
o Answer 5 is incorrect because it is comprised of either two triangles which are
not isosceles (and are not quarters of the square) or only one isosceles triangle
which is also larger than a quarter of the square.

153. Explanation
The correct answer is 3.

In this question you can see that all figures consists of three shapes that are identical
but in three different sizes.

The cloud shape in answer choice 3 is repeated three times in three different sizes.
Therefore, it is the correct answer.

Answer choice 1 is incorrect because the big sized square is repeated twice and
therefore the figure has only two sizes.
Answer choices 2 and 4 are incorrect because the shape is repeated three times in the
same size.
Answer choice 5 is incorrect because it consists of only two shapes.

154. Explanation
The correct answer is 3.

All three figures in this question contain:


• An outer shape which is a square
• An inner shape whose top part is textured, and
• A background whose bottom part is textured

The only answer choice which follows all of these rules is the third choice.

The first choice is incorrect because the textured parts are the bottom of the inner
shape and the top of the background instead of the other way around.
The second choice is incorrect because the outer shape is a circle instead of a square.
The fourth choice is incorrect because both parts of the inner shape are textured, and
the background has no textured part at all.

57
The fifth choice is incorrect because there is no inner shape, and the bottom part of
the background has 2 different textures. (an alternative explanation is that the inner
shape is a rectangle and it is fully textured, instead of just the top half.)
Therefore, the 3rd choice is the correct answer.

155. Explanation
The correct answer is 5.

In this question, we can see that:


All three figures are comprised of two shapes.
One of the shapes is located in the front and the other shape in the back.
The shape in the back has one side more than the shape in the front.

We can see that in the 5th answer choice, the shape in the back has six sides, while
the shape in the front has five sides. This means that the shape in the back has one
side more than the shape in the front.

Therefore, the 5th answer choice is the correct answer.

156. Explanation
The correct answer is 1.
In this question, each picture presents a line spiraling at right angles with black and
white triangles along the line. The pattern is as follows:
 Each spiral begins (or ends) in the top left corner and spirals for three full
revolutions (12 lines).
 The white triangles are all pointing along the line toward the inside of the
spiral, and the black triangles are all pointing toward the outside of the spiral.
 Notice that the number of triangles or their location along the twisted line does
not matter; the only important thing is to make sure all white triangles are
pointing inwards, and all black triangles are pointing outwards.
The only answer that matches the pattern is the 1st choice, and therefore, it is the
correct answer.
o Answer 2 is incorrect because all the triangles in it are pointing outward,
including the white ones.
o Answer 3 is incorrect because the end of the spiral is on the bottom right
instead of the top left (the figure is rotated).
o Answer 4 is incorrect because the black triangles are pointing inwards, and the
white triangles are pointing outwards instead of the other way around.
o Answer 5 is incorrect because the most internal white triangle is pointing
outwards instead of inwards.

157. Explanation
The correct answer is 4.

58
In this question you can see that all figures have four arrows—two are pointing up
and two are pointing down. Additionally, you can see that all figures consist of a
closed outer shape but there is no common rule regarding the outer shape’s features.
Since all the answer choices also consist of a closed outer shape, you should disregard
this rule and take into consideration only the arrows when looking for the correct
answer.

Answer choice 4 has two arrows pointing up and two arrows pointing down and
therefore it is the correct answer.

Answer choice 1 is incorrect because all four arrows are pointing down.
Answer choice 2 is incorrect because it has three arrows pointing up and one arrow
pointing down.
Answer choice 3 is incorrect because it has only three arrows.
Answer choice 5 is incorrect because it has one arrow pointing up and three arrows
pointing down.

158. Explanation
The correct answer is 4.

In this question you can see that all figures have four arrows—two are pointing up
and two are pointing down. Additionally, you can see that all figures consist of a
closed outer shape but there is no common rule regarding the outer shape’s features.
Since all the answer choices also consist of a closed outer shape, you should disregard
this rule and take into consideration only the arrows when looking for the correct
answer.

Answer choice 4 has two arrows pointing up and two arrows pointing down and
therefore it is the correct answer.

Answer choice 1 is incorrect because all four arrows are pointing down.
Answer choice 2 is incorrect because it has three arrows pointing up and one arrow
pointing down.
Answer choice 3 is incorrect because it has only three arrows.
Answer choice 5 is incorrect because it has one arrow pointing up and three arrows
pointing down.

159. Explanation
The correct answer is 2.

In this question, we can see that all three figures consist of an outer circle with two
shapes inside. The outer circle is filled with light blue that fades out from the
upper-right to the lower-left. The only answer choice that follows this pattern is the
2nd one.

59
Therefore, the 2nd figure is the correct answer.

160. Explanation
The correct answer is 3.

In this question, we can see that:


 All three figures consist of an outer shape with a smaller inner shape. The
smaller inner shape is similar to the outer shape.
 All shapes are comprised of straight lines only.
 The outer shape is colored with light blue that fades out from bottom to top.

The 1st choice is incorrect as it is comprised of curved lines.


The 2nd choice is incorrect as it is also comprised of curved lines.
The 4th choice is incorrect as the outer shape is colored with light blue that fades out
from right to left.
The 5th choice is incorrect as the inner shape is colored instead of the outer shape.

Therefore, the 3rd choice is the correct answer.

161. Explanation
The correct answer is 5.

In this question, we can see that all three figures in the top row consist of three
identical shapes that share a common area:

60
Similarly, in the 5th answer choice, we can see that all three circles share a common
area:

The rest of the answer choices do not contain an area that is shared by all three shapes.

Therefore, the 5th answer choice is the correct answer.

162. Explanation
The correct answer is 1.

In this question you can see that:


- All figures include a black outer shape and three white inner smaller shapes.
- In all figures the outer shape has straight sides and sharp corners.
- In all figures there are three inner shapes; two of them have straight sides and sharp
corners, and one is a full circle.

Answer choice 1 has an outer shape with straight sides and sharp corners, and its inner
shapes consists of two shapes that have straight sides and sharp corners, and one full
circle. Therefore, it is the correct answer.

Answer choices 2 and 4 can be eliminated immediately since their outer shape is a
circle.
Answer choice 3 has two inner circles while all the figures on top consist of only one
circle. Therefore, you can eliminate this answer as well.
Now you are left with answers 1 and 5, and you need to find another rule to get to the
correct answer. You can see that there are two differences between the two answers:
1. Answer 1 has two identical inner shapes while in answer 5 all the inner shapes
are different.
2. Answer 1 has a full circle while answer 5 has a ring-shaped circle.

61
When you compare the figures in the answers again to the three figures on top, you
can see that the circle in all three figures on top is a full circle and there is no
ring-shaped circle in any of them. Therefore, you can also eliminate answer 5, and
you are left with answer 1.

163. Explanation
The correct answer is 2.

In this question you can see that:


- All figures consist of three shapes: one black circle and two identical, big, white
shapes, when one of them covers part of the other.
- The shape that covers part of the other shape is the shape that is closer to the circle.

In answer choice 2, the shape that covers the other shape is located next to the circle.
Therefore, this is the correct answer.

Answer choices 1 and 4 are incorrect because the shape that covers the other is further
away from the circle.
Answer choices 3 and 5 are incorrect because both shapes are the same distance from
the circle.

164. Explanation
The correct answer is 3.

In this question you can see that:

- All figures are quadrilateral (they all have four straight sides and four sharp
corners).
- In each figure there is one corner shaded blue. Answer choice 3 is a quadrilateral
and its top left corner is shaded. Therefore, it is the correct answer.

Answer choices 1, 2, and 4 can be eliminated because they are not quadrilaterals
(answer choice 1 has no straight sides, answer choice 2 has rounded corners—not
sharp ones, and answer choice 4 has 5 sides).

Now you are left with answer choices 3 and 5, and should look for another rule in
order to find the correct answer. You can see that the difference between these answer
choices is the shading. In answer 3 the shading is in one of the corners, and in answer
5 the shading is along the side. When looking back at the three top figures, you can
see that in all of them the shading is on a corner. Therefore, you can eliminate answer
choice 5 as well.

62
165. Explanation
The correct answer is 4.

In this question you can see that:


- All figures consist of three different shapes: a triangle pointing down, a square, and
a circle.
- Each shape is filled differently and consistently: the triangle is white, the square is
black, and the circle is checkered.

Answer choice 4 consists of a white triangle pointing down, a checkered circle, and a
black square. Therefore, this is the correct answer.

You can eliminate answers 1, 3, and 5 immediately as the triangle is pointing up


instead of down.
You are now left with answers 2 and 4. In answer 2, the square is checkered and the
circle is black, while in each of the top figures the circle is checkered and the square is
black. Therefore, answer 2 is incorrect and you are left with answer 4.

166. Explanation
The correct answer is 2.
In this question, each picture presents a white figure with several straight lines
protruding from it. The pattern is as follows:
 The number of lines protruding from each figure is equal to twice the number
of sides of the figure.
 For example, the left figure has six sides, so there are twelve lines protruding
from it; the middle figure has 4 sides and 8 lines protruding from it.
The correct answer is the 2nd choice because it contains a triangle, which has three
sides, and six lines are protruding from i

167. Explanation
The correct answer is 2.

In this question, we can see that:


 All three figures contain a shape with two inner lines that meet at a single
point.
 One of the inner lines is straight, while the other is curved.

The 1st answer choice is incorrect as the two inner lines are straight.
The 3rd answer choice is incorrect as this shape contains three inner lines.
The 4th answer choice is incorrect as the two inner lines inside the shape do not meet
at any point.
The 5th answer choice is incorrect as the two inner lines are curved.

In the 2nd answer choice, we can see a shape containing two inner lines that meet at

63
a single point. One of the lines is straight and the other is curved.

Therefore, the 2nd answer choice is the correct answer.

168. Explanation
The correct answer is 5.
In this question, the pattern is as follows:
 All pictures contain two big white figures connected to one another.
 Inside each figure, there is a small grey figure, identical to the other figure in
that picture. Each grey shape is a smaller version of the shape that is
connected to the figure that it is inside of.
 Each small grey figure is in the exact same orientation as its bigger identical
figure.
 Notice that the grey shapes are identical to the white shapes as they are seen,
and each shape does not continue as if the other shape is covering it; one ends,
and the other begins.
The only answer that matches the pattern is the 5th choice, and therefore, it is the
correct answer.
o Answer 1 is incorrect because the bottom small grey figure is different from
the bigger white figure that it is supposed to be identical to.
o Answer 2 is incorrect because the orientation of the small grey triangle is
different from the bigger identical triangle (the triangle is pointing in the
opposite direction).
o Answer 3 is incorrect because the grey figure inside the big triangle is not
identical to the other figure – it is identical to the big figure created from the
two linked figures together.
o Answer 4 is incorrect because the grey figures are identical to the big figure
they are inside of and not the other one.

169. Explanation
The correct answer is 2.

In this question, we can see that:


 All three figures contain a shape with an inner arrow pointing to one of the
shape’s corners.
 In each of the figures, one of the shapes (either the inner arrow or the outer
shape) is white, while the other shape is dark blue.

In the 2nd answer choice, we can see a shape (triangle) with an inner arrow pointing
to one of its corners. In addition, the triangle is dark blue, while the arrow inside it
is white.

Therefore, the 2nd answer choice is the correct answer.

64
170. Explanation
The correct answer is 5.
In this question, the pattern is as follows:
 Each picture contains one small figure inside of a bigger figure.
 The internal figure is comprised of two lines more than the external figure.
 For example, in the left picture, the external figure (a pentagon) is comprised
of five lines, and therefore, the internal figure (the arrow) is comprised of
seven lines.
o Answers 1 and 2 are incorrect because the internal figures inside of them are
comprised of only one (instead of two) more line than the external figures.
o Answers 3 and 4 are incorrect because the internal figures inside of them are
comprised of fewer (instead of more) lines than the external figures.
We are left with the 5th choice as the only possible correct answer, and it is correct
because the external figure is comprised of three lines, and therefore, the internal
figure is comprised of five lines (exactly two more lines).

171. Explanation
The correct answer is 4.
In this question, the pattern is that each of the pictures contain the exact same figure,

in its exact size and orientation. The repeated figure is:


That means that the same figure is repeated in each picture, so the answer should also
contain this exact figure.

The only answer containing the repeated figure is the 5th choice, and therefore, it is
the correct answer.

65
2. Figure Matrices
1. Explanation
The correct answer is 1.

In the top row we have two figures - how do they go together?


We can see that the two figures look similar, but they some changes have occurred
from the left side to the right:
• The outer shape has received one extra side, making it into a new shape
• The middle shape's rounded side has become straight sides with rounded corners
• The inner shape has been flipped on its side
• All the shapes have been closed (the missing side has been added without further
changing the shape)

In the bottom row, we have a single figure, and we need to find a figure that will go
together with it in the same way that the two figures above go together.

The only three choices that change the outer shape to one with one more side are 1, 4,
and 5.
Answer choice 1 makes all four changes listed above.
Answer choice 4 does not close the shapes.
Answer choice 5 does not flip the inner shape on its side.

Therefore, the 1st answer choice is the correct answer.

2. Explanation
The correct answer is 4.

In the top row we have two figures - how do they go together?

We can see that the two figures look similar, but:


• On the right, the figure is flipped horizontally,
• The long vertical line has been removed,
• The short vertical line now runs through entire length of the shape, and -
• A new horizontal line has been added.

In the bottom row we have a single figure, and we need to find a figure that will go
together with it in the same way that the two figures above go together.

We can immediately eliminate answer choice 3 because it looks like the shape in the
top row, and not the bottom row.
We can also eliminate answer choices 1 and 5, because they have two vertical lines,
while we only need one.

66
When we look at the other two answer choices, the only one that matches the pattern
is answer choice 4.
In answer choice 2 there are two horizontal lines while we need only one.

Therefore the 4th answer choice is the correct answer.

3. Explanation
The correct answer is 2.

The relationship between the first two figures is as follows: the second figure is the
same as the first figure, but rotated 90° counterclockwise, and with a triangle
added on top. The correct answer must have the same relationship with the third
figure.

This means that 2 is the correct answer.

4. Explanation
The correct answer is 2.

Look closely at the top row: As you move across the row, the star is doubled and
is partially overlapping. The parts which overlap become black.

Apply the same rule to the bottom row. As you double and partially overlap the
frames, only segments of the top and bottom of the frames will overlap and therefore
become black.

The only answer choice which follows this rule is 2.

5. Explanation
The correct answer is 4.

Take a look at the above image:


Notice that the shapes that have the same frame correspond with each other and move
in a similar way across the rows:

67
•The red rectangle on the left of the box moves to the right, adjacent to the wall of
the box,
•The blue rectangle on the right of the box moves to the left, and
•The green rectangle moves from behind the blue rectangle to behind the red
rectangle.

The same pattern should occur in the bottom row. The only answer choice that
follows these rules is the 4th.

6. Explanation
The correct answer is 1.

Take a look at the top row. The order of the shapes in the left frame determines the
position of the shapes in the right frames.
 The top shape (the star) is positioned in the center,
 The middle shape (the circle) is positioned on the outside, and -
 The bottom shape (the triangle) is positioned in between the two objects.

The same rule will apply in the bottom row:


The circle sector will be in the center, so we can eliminate answer choices 4 and 5.
The plaque will be the outer shape, so we can eliminate answer choice 2.
The ring will be positioned in between both shapes. We can eliminate answer choice 3,
and are left with answer choice 1 as the correct answer.

7. Explanation
The correct answer is 1.

In the top row, when we move from the left box to the right box, the diagonal
lines are removed and the vertical and horizontal lines remain.

When we look at the answer choices to find the one which will have the same
relationship with the left box in the bottom row, the only answer which follows these
rules is the 1st.

8. Explanation
The correct answer is 2.

In the top row we have two figures - how do they go together?

• The shapes in the right figure overlap in the opposite order than they do in the left
figure: on the left, the lower shape always covers the higher shape, and on the right,
the higher shape always covers the lower shape.

68
• Also, the shape in front on the left and the shape in front on the right are always
the same color, while the other two shapes switch colors with each other.

In the bottom row we have a single figure, and we need to find a figure that will go
together with it in the same way that the two figures above go together.

In answer choices 1, 3 and 5 the shape at the front is not gray, so these choices can be
eliminated.
In answer choice 4 the black and white shapes have remained in place, so we can
eliminate it as well.

We are left with answer choice 2, which is the correct answer.

9. Explanation
The correct answer is 3.

In the top row we have two figures - how do they go together?


As we move from left to right:
• The shapes change from triangles to circles,
• The number of shapes and their position remain the same, yet -
• All the shapes but one change color from white to black.

In the bottom row we have a single figure, and we need to find a figure that will go
together with it in the same way that the two figures above go together.

In answer choice 5 we have three circles and not two, so it can be eliminated.
In answer choices 1 and 4 none of the circles have changed color, so we can eliminate
them as well.
In answer choice 2 the shapes have changed position, so it is not a correct answer
either.

We are left with answer choice 3, which is the correct answer.

10. Explanation
The correct answer is 5.

In the top row we have two figures - how do they go together?


As we move from left to right:
• The shapes on the right mirror the shapes on the left,
• The circle moves in front of the triangle,
• The background color changes from gray to black,

69
• The triangle changes from black to gray, and –
• The circle changes from gray to white.

In the bottom row we have a single figure, and we need to find a figure that will go
together with it in the same way that the two figures above go together.

Answer choice 2 can be eliminated because the background hasn’t changed to black.
Answer choice 3 can be eliminated because the circle remained at the back of the
triangle.
Answer choice 4 can be eliminated because the circle has move higher than in the
figure on the left.
Answer choice 1 can be eliminated because the square has moved to the back of the
triangle while it should have remained in place.

Answer choice 5 is the only one that follows all of the above rules and is the correct
answer.

11. Explanation
The correct answer is 1.

In the top row, from left to right, the figure changes as follows:

 A mirror image of the pentagon is added, forming an upside down pentagon


attached to the original one from below.
 The original pentagon changes its color from white to black.

In the bottom row, we should have the same relationship: A mirror image of the
triangle should be added to form an upside down triangle. This upside down triangle
should be attached to the original triangle from below. In addition, the original
triangle should change its color from white to black. Therefore, the 1st choice is the
correct answer.

The 5th choice is incorrect as the correct relationship between the figures does not
involve flipping the original figure from right to left

12. Explanation
The correct answer is 2.

In the top row, from left to right, the figure changes as follows:

•The top shape (circle) becomes the middle shape in the second figure.
•The middle shape (pentagon) becomes the outermost shape in the second figure.
•The bottom shape (arrow) rotates 90° counterclockwise, changes its color to blue,
and becomes the innermost shape in the second figure.

70
In the bottom row, there should be the same relationship:

•The top shape (circle) should become the middle shape.


•The middle shape (pentagon) should become the outermost shape.
•The bottom shape (moon) should be rotated 90° counterclockwise, colored in
with blue, and become the innermost shape.

The second answer choice follows all the above conditions. Therefore, this is the
correct answer.

The third answer choice is incorrect as the moon inside this figure is rotated 90°
clockwise, instead of 90° counterclockwise.

Alternatively, the bottom shape rotates 90 degrees counterclockwise, changes to blue,


and then moves inside the top shape.
Next, the top shapes move into the middle shape.

13. Explanation
The correct answer is 5.

In this question, from left to right, we can see that:

 In the top row, the left-top shape (the smiley face) is flipped over
upside-down but stays in its place. Thus, in the bottom row, the left-top
shape (the heart) should also be flipped over upside-down but stay in its place.
 In the top row, the left-bottom shape (the arrow) stays in its place and does
not change at all. Thus, in the bottom row, the left-bottom shape (the
trapezoid) should also stay in its place.
 In the top row, the right-top shape (the “L-shape”) and the right-bottom shape
(the triangle) switch places. Additionally, the new right-top shape (the
triangle) changes its color from white to black. Thus, in the bottom row, the
right-top shape (the star) and the right-bottom shape (the arc) should switch
places, and the new right-top shape (the arc) should change its color from
white to black.

The 5th answer choice follows all the above conditions. Therefore, this is the correct
answer.

14. Explanation
The correct answer is 3.

In this question, from left to right, we can see that:

71
 In the top row, shapes are inserted into the outer shape (triangle) to create the
figure on the right. The number of these inner shapes is the same as
the number of the outer shape’s sides: In the top row, the outer shape
has three sides, and thus three shapes are inserted. In the bottom row, the
outer shape has four sides, and thus four shapes should be inserted.
 In the top row, each of the newly inserted shapes has one side more than the
outer shape. The outer shape (triangle) has three sides, and each of the inserted
shapes has four sides (squares). In the bottom row, the outer shape (square)
has four sides, and thus each of the inserted shapes should have five sides
(pentagons).
 The inserted shapes are colored with blue.

The 3rd answer choice follows all the above conditions. Therefore, this is the correct
answer.

15. Explanation
The correct answer is (A).
Identify the image that completes the pattern by examining how the series of shapes
change across the rows and down the columns within the matrix. The direction in
which you examine the shapes depends on where you can most easily visualize the
relationship between the shapes.
The outer shape in each row remains the same, however it rotates 90-degrees
clockwise. Therefore, the shape that will appear in the missing box will be a pentagon,
and it will be pointing upwards (in order to match the rotation pattern).
In addition, in each row and column, the shapes are filled with a different number of
black dots. The top two rows have shapes with one, two and three dots. As the third
row includes only shapes with two and three dots, the missing box must have a shape
with one dot.
The only answer that follows those conditions is answer (A).
Answers (B) and (C) contain the correct shape with the correct number of dots on the
inside (one). However, the pentagons they contain are not a 90-degree clockwise
rotation of the box to the left, and therefore they are incorrect.
Answers (D) and (E) have three and two dots inside, respectively. Since the column
and row already contain shapes with three and two dots, these answer choices are
incorrect and can also be ruled out.

16. Explanation
The correct answer is (B).
Identify the image that completes the pattern by examining how the series of shapes
change across the rows and down the columns within the matrix. The direction in
which you examine the shapes depends on where you can most easily visualize the
relationship between the shapes.

72
Look at the first row and try to find a connection between the triangles and the square.
Notice that the square on the right can be created by putting the two triangles together
and removing the diagonal line on the side they share:

The same connection can be applied to the second row as the two triangles can form
the parallelogram on the right. Therefore, the missing box will contain a shape
comprised of the two shapes on the other two boxes. In addition, the shapes in each
column have the same amount of edges which means the empty box must contain a
shape with four edges (like the square and the parallelogram).
The correct answer is (B) as the triangles in this row combined can form:

17. Explanation
The correct answer is (B).
Identify the image that completes the pattern by examining how the series of shapes
change across the rows and down the columns within the matrix. The direction in
which you examine the shapes depends on where you can most easily visualize the
relationship between the shapes.
Each box contains a different geometrical shape. Along the columns, the shapes
number of sides increases by one. For example, in the left column the shape changes
from a triangle to a square and then a pentagon; 3, 4, and 5 sides, respectively. This
rule is enough to answer the question because the answer must have one more side
than the octagon, making the correct answer (B).
Side note: You can also notice that the number of sides in the rows increases by 2.

18. Explanation
The correct answer is (D).
Identify the image that completes the pattern by examining how the series of shapes
change across the rows and down the columns within the matrix. You can try and find
the pattern formed along the rows or down the columns as they both results the same
answer:
Along the rows - the boxes on the right column contain white areas in places that the
two other boxes on the same row contain white areas.
Down the columns - the boxes on the bottom row contain black areas in places that
the two other boxes on the same column contain black areas.
Using either of this methods will result the answer (D).

73
19. Explanation
The correct answer is (D).
Identify the image that completes the pattern by examining how the series of shapes
change across the rows and down the columns within the matrix. The direction in
which you examine the shapes depends on where you can most easily visualize the
relationship between the shapes.
Each row has the same shape with a different number of white dots. When moving
from left to right, the number of dots is divided by 2 (notice the number of dots is also
divided in each column when moving from top to bottom).
Therefore, the empty box must contain an arrow with exactly one dot (as two divided
by two is one), making answer (D) correct.
Side note: the location of the dots is random and only meant to distract from the
pattern.

20. Explanation
The correct answer is (C).
Identify the image that completes the pattern by examining how the series of shapes
change across the rows and down the columns within the matrix. The direction in
which you examine the shapes depends on where you can most easily visualize the
relationship between the shapes.
The first thing to notice is that each row contains a different pair of shapes. Therefore,
you can rule out answer (A) immediately, as it contains ellipses and not stars.
In each row, when moving from left to right the shapes move closer to each other,
while in the rightmost box the shared area is colored black.
The only answer choice that follows this pattern is answer (C), and therefore it is the
correct answer.
In answer (B) everything is colored except the shared area, thus it is incorrect.
In answers (D) and (E) nothing is colored; thus, they are also incorrect.

21. Explanation
The correct answer is (E).
Identify the image that completes the pattern by examining how the series of shapes
change across the rows and down the columns within the matrix. The direction in
which you examine the shapes depends on where you can most easily visualize the
relationship between the shapes.
The left most box in each row is comprised of the other boxes in the same row.
Similarly, the top box in each column is comprised of the other two boxes in that
column.
Therefore, the empty box must contain a shape which, together with the adjacent
boxes, will complete both the shapes in the top box of its column and the leftmost box
of its row. Answer (E) is the correct answer.

22. Explanation
The correct answer is (C).

74
Identify the image that completes the pattern by examining how the series of shapes
change across the rows and down the columns within the matrix. The direction in
which you examine the shapes depends on where you can most easily visualize the
relationship between the shapes.
Across each row the shape in the middle column is comprised from the objects which
appear in only one of the shapes adjacent to it (in the same row). Lines that appear in
both adjacent shapes are not included.
Similarly, in each column the shape in the middle row is comprised of the objects that
appear in only one of the shapes adjacent to it (in the same column). Lines that appear
in both adjacent shapes are not included.
Therefore, the correct answer is (C), as it is the only answer that completes the
pattern.

23. Explanation
The correct answer is 5.
In the top row, from left to right, the figure rotates and the arrow and circle swap
colors.
In the bottom row we should have the same relationship. In the left frame the arrow is
behind the circle. We can eliminate the 1st and 4th answer choices because they have
the arrow in front of the circle. We can eliminate the 2nd choice because the arrow
did not rotate, and the 3rd choice because the shapes did not swap colors.
We are left with the 5th choice as the correct answer.

24. Explanation
The correct answer is 3.

In the top row, from left to right, the number of black shapes swaps with
the number of circles (the two shapes and three circles in the left frame become three
shapes and two circles in the right frame).
In the bottom row we should have the same relationship. In the left frame we have
two circles, therefore in the right frame we should have only two shapes.
We can eliminate the 2nd and 5th answer choices.
In the left frame we have three shapes, therefore in the right frame we should have
three circles.
We can further eliminate the 1st and 4th choices.
We are left with the 3rd choice as the correct answer.

25. Explanation
The correct answer is 4.
In the top row, every arrow in the left frame goes with a particular figure in the right
frame.
 Arrow pointing right = white heart
 Arrow pointing down = grey smiley face
 Arrow pointing up = grey sun

75
In the bottom row we should have the same relationship. We can eliminate the 3rd
answer choice because it contains a grey heart and not a white heart.
We can also eliminate the 1st and 5th choices because they contain frowny faces and
not smiley faces.
Finally we can eliminate the 2nd answer choice because we should have a smiley face
in the upper-right corner (where there was a down arrow in the left frame), and a sun
in the upper-left and lower-right corners (where in the left frame we had up arrows),
but we have the opposite.
We are left with the 4th choice as the only correct answer.

26. Explanation
The correct answer is 1.
There are two changes that occur between the first two figures in the first part of the
analogy: the figure rotates clockwise 90°, and the color changes from black to
white.
The same changes must apply to the third figure to complete the analogy.
This means that answer 1 is the correct answer.

27. Explanation
The correct answer is 3.
The relationship between the first two figures is as follows: from the first frame to the
second frame, all the circles become rectangles, and one of the small rectangles
disappears.
The correct answer must have the same relationship with the third figure: it should be
a large rectangle with three small rectangles.
This means that 3 is the correct answer.

28. Explanation
The correct answer is 2.
The relationship between the first two figures is as follows:
 The bottom shape (trapezoid) in the first figure becomes the outermost
shape in the second figure;
 The middle shape (triangle) is rotated 90° clockwise and becomes
the innermost shape the second figure;
 The top shape (circle) becomes the middle shape in the second figure.
The correct answer should have the same relationship with the third figure.
The bottom shape (rectangle) in the third figure must become the outermost
shape. This means we can eliminate answers 4 and 5.
The middle shape (triangle) must be rotated 90° and become the innermost
shape, and the top shape (circle) must become the middle shape. Answers 1
and 3 can be eliminated, since they do not have the triangle in the correct
position, leaving 2 as the correct answer.

76
29. Explanation
The correct answer is 4.
The relationship between the first two figures is as follows:
- The inner shape (triangle) in the first figure moves to fill in the missing piece of
the outer shape.
- The inner shape (triangle) is rotated 90° counterclockwise.
- The inner shape (triangle) changes from white to black.
The correct answer will have the same relationship with the second figure. Since the
inner shape must move to the outside of the outer shape, we can eliminate answers 1
and 5. Answers 2 and 3 can also be eliminated since the outer shape is not supposed to
rotate.
This leaves 4 as the correct answer.

30. Explanation
The correct answer is 5.
The second figure is the same as first figure with two changes: the figure
becomes shaded, and the figure is extended in the direction of the dark area. The
correct answer should have the same relationship with the third figure.
This means that 5 is the correct answer.

31. Explanation
The correct answer is 2.
The relationship between the first two figures is as follows: the shapes in the first
frame each become shapes with one less side, the black shape becomes white and
the white shape becomes black. The correct answer must have the same relationship
with the third figure. The correct answer will be a large black triangle with a small
white four-sided shape.
This means that 2 is the correct answer.

32. Explanation
The correct answer is 4.
The relationship between the first two figures is as follows: the shapes in the first
frame each become shapes with one more side, the black shape becomes white and
the white shape becomes black. The correct answer must have the same relationship
with the third figure. The correct answer will be a large white pentagon with a small
black four-sided shape.
This means that 4 is the correct answer.

33. Explanation
The correct answer is 1.
The relationship between the first and second figure is as follows: the shape on the
left disappears, and the remaining shape on the right turns black. The correct
answer must have the same relationship with the third figure. This means that the
triangle on the left will disappear, and the remaining rectangle on the right will remain

77
and turn black.
This means that 1 is the correct answer.

34. Explanation
The correct answer is 3.
In the top row, from left to right, the lower half of the figure is removed. The same
should happen in the bottom row.
Since in the left frame we have two grey circles on top and two black circles on the
bottom, in the right frame we should only have the grey circles on top. We can
eliminate all the answer choices with black circles on the bottom (the 1st, 2nd, 4th,
and 5th choices).
We are left with the 3rd choice, which is the only correct answer

35. Explanation
The correct answer is 4.
In the top row, from left to right, the figure rotates 180° (one half of a circle – as if
standing on its head).
In the bottom row we should have the same relationship. We can eliminate the 2nd
and 3rd answer choices because the figure only rotated 90° (a quarter circle – as if
turned on its side). We can also eliminate the 1st and 5th choices because the
rectangles in the center of the figure are both the same color while they should be
different colors (one grey and one white).
We are left with the 4th choice as the only correct answer.

36. Explanation
The correct answer is 3.
Look closely at the top row from left to right:
In the left frame there is a circle with a vertical and a horizontal line inside. In the
next frame (on the right), the circle shrinks and, in the center, overlaps the lines.
The same pattern should occur in the bottom row:
In the left frame, there is a diamond with two diagonal lines. In the next frame, the
diamond should shrink and overlap the diagonal lines in the center of the frame.
Answer choice 1 is incorrect as the diamond is transparent and the lines can be seen.
Answer choice 2 is incorrect since the lines are no longer diagonal and the shape in
the middle is a circle instead of a diamond.
Answer choice 4 is incorrect since the lines are not diagonal and there is no shape in
the center.
Answer choices 5 is incorrect as it contains a black (not grey) diamond.
We are left with the 3rd choice, which is the only correct answer.

37. Explanation
The correct answer is 1.
Take a look at the top row. The order of the shapes in the left frame determines the
position of the shapes in the right frames.

78
 The top shape (the star) is positioned in the center,
 The middle shape (the circle) is positioned on the outside, and -
 The bottom shape (the triangle) is positioned in between the two objects.
The same rule will apply in the bottom row:
The circle sector will be in the center, so we can eliminate answer choices 4 and 5.
The plaque will be the outer shape, so we can eliminate answer choice 2.
The ring will be positioned in between both shapes. We can eliminate answer choice 3,
and are left with answer choice 1 as the correct answer.

38. Explanation
The correct answer is 2.

There are two rules connecting the two shapes. In the top row, from left to right:
1.The shape on the right has been rotated 180 degrees.
2.The shading has changed on the checked part of the shape. In the shape on the right,
the checked pattern, which was on the left shape, has been replaced by light blue
shading.

You need to look for a shape that follows these two rules for the pyramid shape.
Answer choices 1 and 3 have not been rotated correctly through 180 degrees, and
therefore they can be eliminated.

Using rule number 2, look at shapes 2, 4, and 5. In the original shape on the second
row, the patterned part was in the middle section, so look for a shape with light blue
shading in the middle section. Only shape 2 matches this. Check that the other shaded
parts have stayed the same. The white is still white and the dark blue is still dark blue.
Therefore, 2 is the correct answer.

39. Explanation
The correct answer is 4.

In the top row we have two figures - how do they go together?


We can see that the two figures look similar, but:
• The figure on the left is rotated on the right,
And two new parts have been added to the figure:
• A rounded dark shape behind it, and -
• A vertical line that runs in the middle of it.

In the bottom row we have a single figure, and we need to find a figure that will go
together with it in the same way that the two figures above go together.

We can immediately eliminate answer choice 2 because it belongs to the family of


figures in the top row, and not the bottom row.

79
We can also eliminate answer choice 3 because it is positioned in the same way as the
first figure, and is not rotated.
When we look at the other three answer choices, the only one that matches the pattern
is answer choice 4.
Notice that in answer choice 5 the vertical line is missing.
In answer choice 1 the dark shape in the back does not reach the middle of the main
shape, as in the top row.

Therefore the 4th answer choice is the correct answer.

40. Explanation
The correct answer is 3.

In the top row we have two figures - how do they go together?


We can see that the two figures look similar, but:
• On the right, the outer shape has received another side,
• The middle shape has been flipped vertically, and –
• The inner shape has become more rectangular.

In the bottom row we have a single figure, and we need to find a figure that will go
together with it in the same way that the two figures above go together.

We can immediately eliminate answer choice 5 because it belongs to the family of


figures in the top row, and not the bottom row.
We can also eliminate answer choice 2, because it has the same number of sides as the
first figure.
When we look at the other three answer choices, the only one that matches the pattern
is answer choice 3.
In answer choices 1 and 4 the figures are closed and not open.

Therefore the 3rd answer choice is the correct answer.

41. Explanation
The correct answer is 5.

In the top row, from left to right, the shapes move to the center of the frame
and swap colors, and the bottom shape flips.
In the bottom row we should have the same relationship.

We can eliminate the 1st and 2nd answer choices because the shapes did not swap
colors. We can eliminate the 4th choice as the upper shape (the bar) did not move to
the center of the frame. Finally we can eliminate the 3rd choice because the lower
shape (the star) did not flip.

80
We are left with the 5th choice as the only correct answer.

42. Explanation
The correct answer is 2.

In the top row, from left to right, the three arrows flip and the hearts swap colors.

In the bottom row we should have the same relationship.


We can eliminate the 4th answer choice because the hearts should be on the side.
We can eliminate the 1st and 3rd choices because the arrows should be pointing
towards the hearts, not away from them.
Finally we can eliminate the 5th answer choice because the hearts are the wrong
colors.

We are left with the 2nd choice as the correct answer.

43. Explanation
The correct answer is 5.

Look carefully at the change in the top row from left to right.
The background changes from grey to white, and each circle changes to a
square with the shaded part switching from left to right, or right to left (but not up
and down).

In the bottom row we should have a similar relationship. We can easily eliminate the
2nd answer choice because it contains circles (not squares), and the 4th choice
because the background is grey (not white). We can also eliminate the 1st choice
because it changed in the opposite way than how it was supposed to (the black section
rotated from its position in the left frame and switched sides from its position in the
frame above, while it should have done the opposite). Finally, we can eliminate the
3rd answer choice because the black sections moved in different directions for each
shape, and didn't follow any specific rule.

We are left with the 5th choice as the only correct answer.

44. Explanation
The correct answer is 2.

In the top row, from left to right, we added a duplicate shape (another arrow that
was flipped 180 degrees) over the original shape and the overlapping area became
black.

In the bottom row we should have the same relationship. Since in the left frame we
have a square ring shape, in the right frame the shape should be doubled, so we can

81
eliminate the 1st answer choice. Further, only the overlapping areas should be black,
so we can also eliminate the 3rd, 4th and 5th choices.

We are left with the 2nd answer choice as the only correct answer.

45. Explanation
The correct answer is 1.

The relationship between the first two figures is as follows: the second figure
represents all of the lines which are shared by the two shapes in the first figure.
Each of the two shapes in the first figure has a diagonal line which is not shared with
the other. These lines disappear in the second figure.
The correct answer must have the same relationship with the third figure.

This means that 1 is the correct answer.

46. Explanation
The correct answer is 3.
In the top row, the left frame contains a circle, two diagonal lines, and a vertical line.
The right frame contains only the diagonal lines.
The same relationship will hold for the bottom row.
The answer choice will contain only the three diagonal lines present in the bottom-left
frame.
Therefore, answer choice 3 is the correct answer.

47. Explanation
The correct answer is 2.
When we move from left to right across:
 In the top row, the color of the background of the frames changes from black
to white. The same change should occur in the bottom row.
 In the top row, the shapes at the bottom (the circle and the square) switch
sides. The same change should occur in the bottom row, and the triangle and
the rhombus should switch sides.
 In the top row, the shape on the bottom right (the circle) turns from grey to
black. In the bottom row the triangle should turn from gray to black.
 In the top row, the shape at the top (the triangle) rotates 180 degrees. The
same change should occur in the bottom row, and the ¾ circle should rotate
180 degrees.
The only answer choice which follows all rules is the 2nd

48. Explanation
The correct answer is 5.
In the top row we have two figures - how do they go together?
The shape on the right is the same as the figure on the left, rotated 180 degrees.

82
In the bottom row we have a single figure, and we need to find a figure that will go
together with it in the same way that the two figures above go together.
The only answer which fills the rule is the 5th.
Therefore, the correct answer is 5.

49. Explanation
The correct answer is 3.
In the top row we have two figures - how do they go together?
As we move from left to right:

• The white triangles change into black diamonds,


• The hearts rotate 180 degrees but remain gray,
• The number of circles increases by one, and -
• The circles change colors from black to white
In the bottom row, we have a single figure, and we need to find a figure that will go
together with it in the same way that the two figures above go together.
We can eliminate answer choice 2 as it contains a white diamond and not a black one.
In addition, this answer choice contains black circles instead of white ones.
We can also eliminate answer choice 1 as it contains a white heart and not a gray one.
We can eliminate answer choice 4 as it contains three circles instead of four.
Finally, we can eliminate answer choice 5 as it contains a heart that has not been
rotated.
We are left with answer choice 3, which is the correct answer.

50. Explanation
The correct answer is 1.
In the top row we have two figures - how do they go together?
As we move from left to right:
• The inner and outer shapes have switched places – on the right the diamond is now
the outer shape and the circle is now the inner shape.
• The inner shapes contain shaded segments in the same places, but the shading has
changed from gray to brick-textured.
• The outer shapes contain shaded segments in opposite places and the shading has
changed from brick-textured to gray.
In the bottom row we have a single figure, and we need to find a figure that will go
together with it in the same way that the two figures above go together.
Answer choices 2, 3 and 5 don’t have shading in all segments of the inner and outer
shapes, so they can be eliminated.
In answer choice 4 the gray segments of the inner shape have remained gray in the
figure on the right, so we can eliminate it as well.
We are left with answer choice 1, which is the correct answer.

83
51. Explanation
The correct answer is 4.
In this question, from left to right, we can see that:

 In the top row, the outer shape (trapezoid) becomes a shape with one less
side (triangle). Thus, in the bottom row, the outer shape (rectangle) should also
change into a shape with one less side (triangle).
 In the top row, the inner shape (pentagon) becomes a shape with one less
side (rectangle). Thus, in the bottom row, the inner shape (hexagon) should
also change into a shape with one less side (a pentagon).
 The color of the outer shape fades out from top to bottom instead of bottom
to top.
 The inner shape changes its color from black to white.
The 4th answer choice follows all the above conditions. Therefore, this is the correct
answer.
The 2nd answer choice is incorrect as the color of the outer shape should fade out
from top to bottom, not from bottom to top.

52. Explanation
The correct answer is 2.
In this question, from left to right, we can see that:

 In the top row, the whole shape (parallelogram) becomes black. Thus, in the
bottom row, the whole shape (heart) should also become black.
 In the top row, a white arrow is added to the right-hand figure. It points in the
direction of where the dark area in the left-hand figure was located (the
right-bottom corner of the parallelogram). Thus, in the bottom row, a white
arrow pointing to where the left-hand figure's dark area was located (the
bottom corner of the heart) should be added to the right-hand figure.
The 2nd answer choice follows all the above conditions. Therefore, this is the correct
answer.

53. Explanation
The correct answer is 2.
In the top row, there are two figures — how do they go together?
It seems that in the figure on the right, only the bottom triangle stays.
Thus, in the bottom row, let’s look for an answer choice that is the same as the bottom
diamond.
We can see that there is no such answer choice. Therefore, we need to think of
another way the two figures in the top row go together.
If we look closely at the right figure in the top row, we can see that it consists of only
the lines shared by both triangles on the left. Both triangles on the left have two
vertical lines. However, the top triangle has one additional horizontal line which the
bottom triangle doesn't have. Thus, in the figure on the right, this horizontal line

84
disappears.

In the bottom row, we should have the same relationship. The correct answer should
consist of only the lines shared by both diamonds on the left. Both diamonds on the
left have one horizontal line. However, the bottom diamond has one additional short
diagonal line which the top diamond doesn't have. Thus, in the figure on the right, this
short diagonal line should disappear.

The 2nd answer choice follows this condition. Therefore, the correct answer is 2.

54. Explanation
The correct answer is (B).
Identify the image that completes the pattern by examining how the series of shapes
change across the rows and down the columns within the matrix. The direction in
which you examine the shapes depends on where you can most easily visualize the
relationship between the shapes.
When you look at each row from left to right, notice the shape lose a line. When
moving from the leftmost box to the middle one, the shape loses its vertical line, and
when moving from the middle box to the rightmost box the shape loses the horizontal
line, leaving only the diagonal lines.
When moving from top to bottom in each column the shapes lose a line. When
moving from the top box to the middle box the shape loses the diagonals which
stretch from top-right to bottom-left, and when moving from the middle box to the
bottom the shape loses the other diagonal (which stretches from top-left to
bottom-right).
Answer (B) is the only answer which follows the pattern (the empty box), and
therefore it is the correct answer.
Alternative Solution: When moving either left to right or top to bottom, the shape
loses exactly one line. Therefore, the answer box must be empty to meet this
condition.

55. Explanation
The correct answer is (A).
Identify the image that completes the pattern by examining how the series of shapes
change across the rows and down the columns within the matrix. The direction in
which you examine the shapes depends on where you can most easily visualize the
relationship between the shapes.
Each column has a different kind of background. In addition, across each row (when
moving from left to right) the position of the black dot changes and moves clockwise
along the corners of the shape.
The correct answer is (A), since it is the only answer choice which follows the
pattern.
Answer choices (B), (C) and (E) can be ruled out as their background is different than
the background shown in the column of the missing box.

85
The dot in answer (D) is misplaced, making this answer choice incorrect.

56. Explanation
The correct answer is (B).
Identify the image that completes the pattern by examining how the series of shapes
change across the rows and down the columns within the matrix. The direction in
which you examine the shapes depends on where you can most easily visualize the
relationship between the shapes.
Across each row the box on the right contains a quotient arrived at by dividing the
other two numbers in the same row.
Along each column the box at the bottom is the product of multiplying the other two
boxes in that column.
Therefore, the answer is (B), as along its row 6 ÷ 2 = 3 and down that column 1 × 3 =
3.

57. Explanation
The correct answer is (C).
Identify the image that completes the pattern by examining how the series of shapes
change across the rows and down the columns within the matrix. The direction in
which you examine the shapes depends on where you can most easily visualize the
relationship between the shapes.
In each row the box on the right sums the number of white circles in the other boxes
in its row. Similarly, the box on the bottom in each column sums the number of black
circles in its column.
Therefore, the empty box should sum the number of black circles in its column (four)
and white circles in its row (four), making answer (E) correct.

58. Explanation
The correct answer is (A).
Identify the image that completes the pattern by examining how the series of shapes
change across the rows and down the columns within the matrix. The direction in
which you examine the shapes depends on where you can most easily visualize the
relationship between the shapes.
Across each row the image is rotated 90-degrees clockwise. Therefore, the empty box
will contain a shape similar to the box on its left, but rotated, making answer (A)
correct.

59. Explanation
The correct answer is (C).
Identify the image that completes the pattern by examining how the series of shapes
change across the rows and down the columns within the matrix. The direction in
which you examine the shapes depends on where you can most easily visualize the
relationship between the shapes.

86
When going through the pictures in each row the shapes rotate counter-clockwise, and
when moving down each column (from top to bottom) the shapes rotate clockwise,
while in the bottom row the stars are deleted.
Answer (C) is the correct answer as it fits the pattern. It does not include stars, the
shapes inside are a clockwise rotated version of the box above it, and a
counter-clockwise rotated version of the box to its left.

60. Explanation
The correct answer is (E).
Identify the image that completes the pattern by examining how the series of shapes
change across the rows and down the columns within the matrix. The direction in
which you examine the shapes depends on where you can most easily visualize the
relationship between the shapes.
Across each row the white star moves from left to right. Down each column the white
star moves from up to down; in the top row it is positioned above the black stars, in
the middle row it is positioned in between them, and in the bottom row it is positioned
below them. The empty box is on the bottom row; therefore, the white star will be
below the black stars and it is also in on the right column, thus the star will appear on
the right. Therefore, the correct answer is (E).

61. Explanation
The correct answer is (D).
Identify the image that completes the pattern by examining how the series of shapes
change across the rows and down the columns within the matrix. The direction in
which you examine the shapes depends on where you can most easily visualize the
relationship between the shapes.
Every box contains the same shapes – a square, a triangle and a circle, and one of
those shapes is patterned with diagonal lines. You can notice that each column has the
exact same shape, thus the empty box will contain a shape similar to its column.
Therefore, you can rule out answers (B), (C) and (E).
In addition, when moving across each row the patterned shape is either the inner,
middle or outer one: The top row has the outer shape colored, the colored shape in the
middle row is the one in the middle, and the bottom row has the innermost colored.
As the empty box is in the last row, the patterned shape will be the inner shape.
Therefore, the correct answer is (D).

62. Explanation
The correct answer is 3.

In the top row we have two figures - how do they go together?


We can see that the two figures look similar, but:

• The figure on the left is flipped horizontally on the right.


• Also, the circle is now dark and has moved behind the shape.

87
Two new shapes have been added to the figure:
• A vertical line next to the dark circle, and -
• A white circle in the corner above it.

In the bottom row, we have a single figure, and we need to find a figure that will go
together with it in the same way that the two figures above go together.

We can immediately eliminate answer choice 4 because it belongs to the family of


figures in the top row, and not the bottom row.
We can also eliminate answer choice 2, because it is positioned in the same way as
the first shape, and is not flipped.
When we look at the other three answer choices, we see that the only one that matches
the pattern is answer choice 3.
Notice that answer choice 5 includes nearly all the changes we need, but it is a closed
shape and not an open one like the figures in the question.

Therefore the 3rd answer choice is the correct answer.

63. Explanation
The correct answer is 1.

In the top row we have two figures - how do they go together?


We can see that the two figures look similar, but they some changes have occurred
from the left side to the right:
• The outer shape has received one extra side, making it into a new shape
• The middle shape's rounded side has become straight sides with rounded corners
• The inner shape has been flipped on its side
• All the shapes have been closed (the missing side has been added without further
changing the shape)

In the bottom row, we have a single figure, and we need to find a figure that will go
together with it in the same way that the two figures above go together.

The only three choices that change the outer shape to one with one more side are 1, 4,
and 5.
Answer choice 1 makes all four changes listed above.
Answer choice 4 does not close the shapes.
Answer choice 5 does not flip the inner shape on its side.

Therefore, the 1st answer choice is the correct answer.

64. Explanation
The correct answer is 4.

88
In the top row we have two figures - how do they go together?

We can see that the two figures look similar, but:


• On the right, the figure is flipped horizontally,
• The long vertical line has been removed,
• The short vertical line now runs through entire length of the shape, and -
• A new horizontal line has been added.

In the bottom row we have a single figure, and we need to find a figure that will go
together with it in the same way that the two figures above go together.

We can immediately eliminate answer choice 3 because it looks like the shape in the
top row, and not the bottom row.
We can also eliminate answer choices 1 and 5, because they have two vertical lines,
while we only need one.
When we look at the other two answer choices, the only one that matches the pattern
is answer choice 4.
In answer choice 2 there are two horizontal lines while we need only one.

Therefore the 4th answer choice is the correct answer.

65. Explanation
The correct answer is 2.

The relationship between the first two figures is as follows: the second figure is the
same as the first figure, but rotated 90° counterclockwise, and with a triangle
added on top. The correct answer must have the same relationship with the third
figure.

This means that 2 is the correct answer.

66. Explanation
The correct answer is 2.

Look closely at the top row: As you move across the row, the star is doubled and
is partially overlapping. The parts which overlap become black.

Apply the same rule to the bottom row. As you double and partially overlap the
frames, only segments of the top and bottom of the frames will overlap and therefore
become black.

The only answer choice which follows this rule is 2.

89
67. Explanation
The correct answer is 4.

Take a look at the above image:


Notice that the shapes that have the same frame correspond with each other and move
in a similar way across the rows:

•The red rectangle on the left of the box moves to the right, adjacent to the wall of
the box,
•The blue rectangle on the right of the box moves to the left, and
•The green rectangle moves from behind the blue rectangle to behind the red
rectangle.

The same pattern should occur in the bottom row. The only answer choice that
follows these rules is the 4th.

68. Explanation
The correct answer is 1.

In the top row, when we move from the left box to the right box, the diagonal
lines are removed and the vertical and horizontal lines remain.

When we look at the answer choices to find the one which will have the same
relationship with the left box in the bottom row, the only answer which follows these
rules is the 1st.

90
69. Explanation
The correct answer is 3.

In the top row we have two figures - how do they go together?


As we move from left to right:
•The shapes change from triangles to circles,
•The number of shapes and their position remain the same,yet -
•All the shapes but one change color from white to black.

In the bottom row we have a single figure, and we need to find a figure that will go
together with it in the same way that the two figures above go together.

In answer choice 5 we have three circles and not two, so it can be eliminated.
In answer choices 1 and 4 none of the circles have changed color, so we can eliminate
them as well.
In answer choice 2 the shapes have changed position, so it is not a correct answer
either.
We are left with answer choice 3, which is the correct answer.

70. Explanation
The correct answer is 5.
In the top row we have two figures - how do they go together?
As we move from left to right:
The shapes on the right mirror the shapes on the left,
The circle moves in front of the triangle,
•The background color changes from gray to black,
•The triangle changes from black to gray, and –
•The circle changes from gray to white.
In the bottom row we have a single figure, and we need to find a figure that will go
together with it in the same way that the two figures above go together.

Answer choice 2 can be eliminated because the background hasn’t changed to black.
Answer choice 3 can be eliminated because the circle remained at the back of the
triangle.
Answer choice 4 can be eliminated because the circle has move higher than in the
figure on the left.
Answer choice 1 can be eliminated because the square has moved to the back of the
triangle while it should have remained in place.

Answer choice 5 is the only one that follows all of the above rules and is the correct
answer.

91
71. Explanation
The correct answer is 1.
In the top row, from left to right, the figure changes as follows:
 A mirror image of the pentagon is added, forming an upside down pentagon
attached to the original one from below.
 The original pentagon changes its color from white to black.

In the bottom row, we should have the same relationship: A mirror image of the
triangle should be added to form an upside down triangle. This upside down triangle
should be attached to the original triangle from below. In addition, the original
triangle should change its color from white to black. Therefore, the 1st choice is the
correct answer.
The 5th choice is incorrect as the correct relationship between the figures does not
involve flipping the original figure from right to left.

72. Explanation
The correct answer is 2.

In the top row, from left to right, the figure changes as follows:

•The top shape (circle) becomes the middle shape in the second figure.
•The middle shape (pentagon) becomes the outermost shape in the second figure.
•The bottom shape (arrow) rotates 90° counterclockwise, changes its color to blue,
and becomes the innermost shape in the second figure.

In the bottom row, there should be the same relationship:

•The top shape (circle) should become the middle shape.


•The middle shape (pentagon) should become the outermost shape.
•The bottom shape (moon) should be rotated 90° counterclockwise, colored in
with blue, and become the innermost shape.

The second answer choice follows all the above conditions. Therefore, this is the
correct answer.

The third answer choice is incorrect as the moon inside this figure is rotated 90°
clockwise, instead of 90° counterclockwise.

Alternatively, the bottom shape rotates 90 degrees counterclockwise, changes to blue,


and then moves inside the top shape.
Next, the top shapes move into the middle shape.

92
73. Explanation
The correct answer is 5.

In this question, from left to right, we can see that:

 In the top row, the left-top shape (the smiley face) is flipped over
upside-down but stays in its place. Thus, in the bottom row, the left-top
shape (the heart) should also be flipped over upside-down but stay in its place.
 In the top row, the left-bottom shape (the arrow) stays in its place and does
not change at all. Thus, in the bottom row, the left-bottom shape (the
trapezoid) should also stay in its place.
 In the top row, the right-top shape (the “L-shape”) and the right-bottom shape
(the triangle) switch places. Additionally, the new right-top shape (the
triangle) changes its color from white to black. Thus, in the bottom row, the
right-top shape (the star) and the right-bottom shape (the arc) should switch
places, and the new right-top shape (the arc) should change its color from
white to black.

The 5th answer choice follows all the above conditions. Therefore, this is the correct
answer.

74. Explanation
The correct answer is 3.
In this question, from left to right, we can see that:
 In the top row, shapes are inserted into the outer shape (triangle) to create the
figure on the right. The number of these inner shapes is the same as
the number of the outer shape’s sides: In the top row, the outer shape
has three sides, and thus three shapes are inserted. In the bottom row, the
outer shape has four sides, and thus four shapes should be inserted.
 In the top row, each of the newly inserted shapes has one side more than the
outer shape. The outer shape (triangle) has three sides, and each of the inserted
shapes has four sides (squares). In the bottom row, the outer shape (square)
has four sides, and thus each of the inserted shapes should have five sides
(pentagons).
 The inserted shapes are colored with blue.
The 3rd answer choice follows all the above conditions. Therefore, this is the
correct answer.

75. Explanation
The correct answer is (A).
Identify the image that completes the pattern by examining how the series of shapes
change across the rows and down the columns within the matrix. The direction in
which you examine the shapes depends on where you can most easily visualize the
relationship between the shapes.

93
The outer shape in each row remains the same, however it rotates 90-degrees
clockwise. Therefore, the shape that will appear in the missing box will be a pentagon,
and it will be pointing upwards (in order to match the rotation pattern).
In addition, in each row and column, the shapes are filled with a different number of
black dots. The top two rows have shapes with one, two and three dots. As the third
row includes only shapes with two and three dots, the missing box must have a shape
with one dot.
The only answer that follows those conditions is answer (A).
Answers (B) and (C) contain the correct shape with the correct number of dots on the
inside (one). However, the pentagons they contain are not a 90-degree clockwise
rotation of the box to the left, and therefore they are incorrect.
Answers (D) and (E) have three and two dots inside, respectively. Since the column
and row already contain shapes with three and two dots, these answer choices are
incorrect and can also be ruled out.

76. Explanation
The correct answer is (B).
Identify the image that completes the pattern by examining how the series of shapes
change across the rows and down the columns within the matrix. The direction in
which you examine the shapes depends on where you can most easily visualize the
relationship between the shapes.
Look at the first row and try to find a connection between the triangles and the square.
Notice that the square on the right can be created by putting the two triangles together
and removing the diagonal line on the side they share:

The same connection can be applied to the second row as the two triangles can form
the parallelogram on the right. Therefore, the missing box will contain a shape
comprised of the two shapes on the other two boxes. In addition, the shapes in each
column have the same amount of edges which means the empty box must contain a
shape with four edges (like the square and the parallelogram).
The correct answer is (B) as the triangles in this row combined can form:

77. Explanation
The correct answer is (B).
Identify the image that completes the pattern by examining how the series of shapes
change across the rows and down the columns within the matrix. The direction in

94
which you examine the shapes depends on where you can most easily visualize the
relationship between the shapes.
Each box contains a different geometrical shape. Along the columns, the shapes
number of sides increases by one. For example, in the left column the shape changes
from a triangle to a square and then a pentagon; 3, 4, and 5 sides, respectively. This
rule is enough to answer the question because the answer must have one more side
than the octagon, making the correct answer (B).
Side note: You can also notice that the number of sides in the rows increases by 2.

78. Explanation
The correct answer is (D).
Identify the image that completes the pattern by examining how the series of shapes
change across the rows and down the columns within the matrix. You can try and find
the pattern formed along the rows or down the columns as they both results the same
answer:
Along the rows - the boxes on the right column contain white areas in places that the
two other boxes on the same row contain white areas.
Down the columns - the boxes on the bottom row contain black areas in places that
the two other boxes on the same column contain black areas.
Using either of this methods will result the answer (D).

79. Explanation
The correct answer is (D).
Identify the image that completes the pattern by examining how the series of shapes
change across the rows and down the columns within the matrix. The direction in
which you examine the shapes depends on where you can most easily visualize the
relationship between the shapes.
Each row has the same shape with a different number of white dots. When moving
from left to right, the number of dots is divided by 2 (notice the number of dots is also
divided in each column when moving from top to bottom).
Therefore, the empty box must contain an arrow with exactly one dot (as two divided
by two is one), making answer (D) correct.
Side note: the location of the dots is random and only meant to distract from the
pattern.

80. Explanation
The correct answer is (C).
Identify the image that completes the pattern by examining how the series of shapes
change across the rows and down the columns within the matrix. The direction in
which you examine the shapes depends on where you can most easily visualize the
relationship between the shapes.
The first thing to notice is that each row contains a different pair of shapes. Therefore,
you can rule out answer (A) immediately, as it contains ellipses and not stars.

95
In each row, when moving from left to right the shapes move closer to each other,
while in the rightmost box the shared area is colored black.
The only answer choice that follows this pattern is answer (C), and therefore it is the
correct answer.
In answer (B) everything is colored except the shared area, thus it is incorrect.
In answers (D) and (E) nothing is colored; thus, they are also incorrect.

81. Explanation
The correct answer is (E).
Identify the image that completes the pattern by examining how the series of shapes
change across the rows and down the columns within the matrix. The direction in
which you examine the shapes depends on where you can most easily visualize the
relationship between the shapes.
The left most box in each row is comprised of the other boxes in the same row.
Similarly, the top box in each column is comprised of the other two boxes in that
column.
Therefore, the empty box must contain a shape which, together with the adjacent
boxes, will complete both the shapes in the top box of its column and the leftmost box
of its row. Answer (E) is the correct answer.

82. Explanation
The correct answer is (C).
Identify the image that completes the pattern by examining how the series of shapes
change across the rows and down the columns within the matrix. The direction in
which you examine the shapes depends on where you can most easily visualize the
relationship between the shapes.
Across each row the shape in the middle column is comprised from the objects which
appear in only one of the shapes adjacent to it (in the same row). Lines that appear in
both adjacent shapes are not included.
Similarly, in each column the shape in the middle row is comprised of the objects that
appear in only one of the shapes adjacent to it (in the same column). Lines that appear
in both adjacent shapes are not included.
Therefore, the correct answer is (C), as it is the only answer that completes the
pattern.
Alternatively, you could look at the column on the right and the bottom row. The
shapes in the column on the right include objects that appear in only one of the shapes
in the same row. If an object appears in more than one other shape, it will not appear
in the shape on the right column.
Likewise, the shapes in the bottom row include objects that appear in only one of the
shapes in that column. If an object appears in more than one other shape, it will not
appear in the shape on the bottom row.

96
83. Explanation
The correct answer is 2.

There are two rules connecting the two shapes. In the top row, from left to right:
1.The shape on the right has been rotated 180 degrees.
2.The shading has changed on the checked part of the shape. In the shape on the right,
the checked pattern, which was on the left shape, has been replaced by light blue
shading.

You need to look for a shape that follows these two rules for the pyramid shape.
Answer choices 1 and 3 have not been rotated correctly through 180 degrees, and
therefore they can be eliminated.

Using rule number 2, look at shapes 2, 4, and 5. In the original shape on the second
row, the patterned part was in the middle section, so look for a shape with light blue
shading in the middle section. Only shape 2 matches this. Check that the other shaded
parts have stayed the same. The white is still white and the dark blue is still dark blue.
Therefore, 2 is the correct answer.

84. Explanation
The correct answer is 3.

In the top row we have two figures - how do they go together?


We can see that the two figures look similar, but:
• On the right, the outer shape has received another side,
• The middle shape has been flipped vertically, and –
• The inner shape has become more rectangular.

In the bottom row we have a single figure, and we need to find a figure that will go
together with it in the same way that the two figures above go together.

We can immediately eliminate answer choice 5 because it belongs to the family of


figures in the top row, and not the bottom row.
We can also eliminate answer choice 2, because it has the same number of sides as the
first figure.
When we look at the other three answer choices, the only one that matches the pattern
is answer choice 3.
In answer choices 1 and 4 the figures are closed and not open.

Therefore the 3rd answer choice is the correct answer.

97
85. Explanation
The correct answer is 5.

In the top row, from left to right, the shapes move to the center of the frame
and swap colors, and the bottom shape flips.
In the bottom row we should have the same relationship.

We can eliminate the 1st and 2nd answer choices because the shapes did not swap
colors. We can eliminate the 4th choice as the upper shape (the bar) did not move to
the center of the frame. Finally we can eliminate the 3rd choice because the lower
shape (the star) did not flip.

We are left with the 5th choice as the only correct answer.

86. Explanation
The correct answer is 2.

In the top row, from left to right, the three arrows flip and the hearts swap colors.

In the bottom row we should have the same relationship.


We can eliminate the 4th answer choice because the hearts should be on the side.
We can eliminate the 1st and 3rd choices because the arrows should be pointing
towards the hearts, not away from them.
Finally we can eliminate the 5th answer choice because the hearts are the wrong
colors.

We are left with the 2nd choice as the correct answer.

87. Explanation
The correct answer is 5.

Look carefully at the change in the top row from left to right.
The background changes from grey to white, and each circle changes to a
square with the shaded part switching from left to right, or right to left (but not up
and down).

In the bottom row we should have a similar relationship. We can easily eliminate the
2nd answer choice because it contains circles (not squares), and the 4th choice
because the background is grey (not white). We can also eliminate the 1st choice
because it changed in the opposite way than how it was supposed to (the black section
rotated from its position in the left frame and switched sides from its position in the
frame above, while it should have done the opposite). Finally, we can eliminate the
3rd answer choice because the black sections moved in different directions for each
shape, and didn't follow any specific rule.

98
We are left with the 5th choice as the only correct answer.

88. Explanation
The correct answer is 2.

In the top row, from left to right, we added a duplicate shape (another arrow that
was flipped 180 degrees) over the original shape and the overlapping area became
black.

In the bottom row we should have the same relationship. Since in the left frame we
have a square ring shape, in the right frame the shape should be doubled, so we can
eliminate the 1st answer choice. Further, only the overlapping areas should be black,
so we can also eliminate the 3rd, 4th and 5th choices.

We are left with the 2nd answer choice as the only correct answer.

89. Explanation
The correct answer is 1.

The relationship between the first two figures is as follows: the second figure
represents all of the lines which are shared by the two shapes in the first figure.
Each of the two shapes in the first figure has a diagonal line which is not shared with
the other. These lines disappear in the second figure.
The correct answer must have the same relationship with the third figure.

This means that 1 is the correct answer.

90. Explanation
The correct answer is 2.

When we move from left to right across:


 In the top row, the color of the background of the frames changes from black
to white. The same change should occur in the bottom row.
 In the top row, the shapes at the bottom (the circle and the square) switch
sides. The same change should occur in the bottom row, and the triangle and
the rhombus should switch sides.
 In the top row, the shape on the bottom right (the circle) turns from grey to
black. In the bottom row the triangle should turn from gray to black.
 In the top row, the shape at the top (the triangle) rotates 180 degrees. The
same change should occur in the bottom row, and the ¾ circle should rotate
180 degrees.
The only answer choice which follows all rules is the 2nd.

99
91. Explanation
The correct answer is 3.

In the top row we have two figures - how do they go together?


As we move from left to right:

• The white triangles change into black diamonds,


• The hearts rotate 180 degrees but remain gray,
• The number of circles increases by one, and -
• The circles change colors from black to white
In the bottom row, we have a single figure, and we need to find a figure that will go
together with it in the same way that the two figures above go together.

We can eliminate answer choice 2 as it contains a white diamond and not a black one.
In addition, this answer choice contains black circles instead of white ones.
We can also eliminate answer choice 1 as it contains a white heart and not a gray one.
We can eliminate answer choice 4 as it contains three circles instead of four.
Finally, we can eliminate answer choice 5 as it contains a heart that has not been
rotated.

We are left with answer choice 3, which is the correct answer.

92. Explanation
The correct answer is 1.

In the top row we have two figures - how do they go together?


As we move from left to right:
• The inner and outer shapes have switched places – on the right the diamond is now
the outer shape and the circle is now the inner shape.
• The inner shapes contain shaded segments in the same places, but the shading has
changed from gray to brick-textured.
• The outer shapes contain shaded segments in opposite places and the shading has
changed from brick-textured to gray.

In the bottom row we have a single figure, and we need to find a figure that will go
together with it in the same way that the two figures above go together.

Answer choices 2, 3 and 5 don’t have shading in all segments of the inner and outer
shapes, so they can be eliminated.
In answer choice 4 the gray segments of the inner shape have remained gray in the
figure on the right, so we can eliminate it as well.

We are left with answer choice 1, which is the correct answer.

100
93. Explanation
The correct answer is 4.

In this question, from left to right, we can see that:

 In the top row, the outer shape (trapezoid) becomes a shape with one less
side (triangle). Thus, in the bottom row, the outer shape (rectangle) should also
change into a shape with one less side (triangle).
 In the top row, the inner shape (pentagon) becomes a shape with one less
side (rectangle). Thus, in the bottom row, the inner shape (hexagon) should
also change into a shape with one less side (a pentagon).
 The color of the outer shape fades out from top to bottom instead of bottom
to top.
 The inner shape changes its color from black to white.

The 4th answer choice follows all the above conditions. Therefore, this is the correct
answer.

The 2nd answer choice is incorrect as the color of the outer shape should fade out
from top to bottom, not from bottom to top.

94. Explanation
The correct answer is 2.

In this question, from left to right, we can see that:

 In the top row, the whole shape (parallelogram) becomes black. Thus, in the
bottom row, the whole shape (heart) should also become black.
 In the top row, a white arrow is added to the right-hand figure. It points in the
direction of where the dark area in the left-hand figure was located (the
right-bottom corner of the parallelogram). Thus, in the bottom row, a white
arrow pointing to where the left-hand figure's dark area was located (the
bottom corner of the heart) should be added to the right-hand figure.
The 2nd answer choice follows all the above conditions. Therefore, this is the correct
answer.

95. Explanation
The correct answer is 2.

In the top row, there are two figures — how do they go together?
It seems that in the figure on the right, only the bottom triangle stays.
Thus, in the bottom row, let’s look for an answer choice that is the same as the bottom
diamond.

101
We can see that there is no such answer choice. Therefore, we need to think of
another way the two figures in the top row go together.

If we look closely at the right figure in the top row, we can see that it consists of only
the lines shared by both triangles on the left. Both triangles on the left have two
vertical lines. However, the top triangle has one additional horizontal line which the
bottom triangle doesn't have. Thus, in the figure on the right, this horizontal line
disappears.

In the bottom row, we should have the same relationship. The correct answer should
consist of only the lines shared by both diamonds on the left. Both diamonds on the
left have one horizontal line. However, the bottom diamond has one additional short
diagonal line which the top diamond doesn't have. Thus, in the figure on the right, this
short diagonal line should disappear.

The 2nd answer choice follows this condition. Therefore, the correct answer is 2.

96. Explanation
The correct answer is (A).
Identify the image that completes the pattern by examining how the series of shapes
change across the rows and down the columns within the matrix. The direction in
which you examine the shapes depends on where you can most easily visualize the
relationship between the shapes.
Each column has a different kind of background. In addition, across each row (when
moving from left to right) the position of the black dot changes and moves clockwise
along the corners of the shape.
The correct answer is (A), since it is the only answer choice which follows the
pattern.
Answer choices (B), (C) and (E) can be ruled out as their background is different than
the background shown in the column of the missing box.
The dot in answer (D) is misplaced, making this answer choice incorrect.

97. Explanation
The correct answer is (C).
Identify the image that completes the pattern by examining how the series of shapes
change across the rows and down the columns within the matrix. The direction in
which you examine the shapes depends on where you can most easily visualize the
relationship between the shapes.
In each row the box on the right sums the number of white circles in the other boxes
in its row. Similarly, the box on the bottom in each column sums the number of black
circles in its column.
Therefore, the empty box should sum the number of black circles in its column (four)
and white circles in its row (four), making answer (E) correct.

102
98. Explanation
The correct answer is (A).
Identify the image that completes the pattern by examining how the series of shapes
change across the rows and down the columns within the matrix. The direction in
which you examine the shapes depends on where you can most easily visualize the
relationship between the shapes.
Across each row the image is rotated 90-degrees clockwise. Therefore, the empty box
will contain a shape similar to the box on its left, but rotated, making answer (A)
correct.

99. Explanation
The correct answer is (C).
Identify the image that completes the pattern by examining how the series of shapes
change across the rows and down the columns within the matrix. The direction in
which you examine the shapes depends on where you can most easily visualize the
relationship between the shapes.
When going through the pictures in each row the shapes rotate counter-clockwise, and
when moving down each column (from top to bottom) the shapes rotate clockwise,
while in the bottom row the stars are deleted.
Answer (C) is the correct answer as it fits the pattern. It does not include stars, the
shapes inside are a clockwise rotated version of the box above it, and a
counter-clockwise rotated version of the box to its left.

100. Explanation
The correct answer is (E).
Identify the image that completes the pattern by examining how the series of shapes
change across the rows and down the columns within the matrix. The direction in
which you examine the shapes depends on where you can most easily visualize the
relationship between the shapes.
Across each row the white star moves from left to right. Down each column the white
star moves from up to down; in the top row it is positioned above the black stars, in
the middle row it is positioned in between them, and in the bottom row it is positioned
below them. The empty box is on the bottom row; therefore, the white star will be
below the black stars and it is also in on the right column, thus the star will appear on
the right. Therefore, the correct answer is (E).

101. Explanation
The correct answer is (D).
Identify the image that completes the pattern by examining how the series of shapes
change across the rows and down the columns within the matrix. The direction in
which you examine the shapes depends on where you can most easily visualize the
relationship between the shapes.
Every box contains the same shapes – a square, a triangle and a circle, and one of
those shapes is patterned with diagonal lines. You can notice that each column has the

103
exact same shape, thus the empty box will contain a shape similar to its column.
Therefore, you can rule out answers (B), (C) and (E).
In addition, when moving across each row the patterned shape is either the inner,
middle or outer one: The top row has the outer shape colored, the colored shape in the
middle row is the one in the middle, and the bottom row has the innermost colored.
As the empty box is in the last row, the patterned shape will be the inner shape.
Therefore, the correct answer is (D).

102. Explanation
The correct answer is 5.

In the top row, from left to right, the figure rotates and the arrow and circle swap
colors.

In the bottom row we should have the same relationship. In the left frame the arrow is
behind the circle. We can eliminate the 1st and 4th answer choices because they have
the arrow in front of the circle. We can eliminate the 2nd choice because the arrow
did not rotate, and the 3rd choice because the shapes did not swap colors.

We are left with the 5th choice as the correct answer.

103. Explanation
The correct answer is 4.
In the top row, every arrow in the left frame goes with a particular figure in the right
frame.
 Arrow pointing right = white heart
 Arrow pointing down = grey smiley face
 Arrow pointing up = grey sun
In the bottom row we should have the same relationship. We can eliminate the 3rd
answer choice because it contains a grey heart and not a white heart.
We can also eliminate the 1st and 5th choices because they contain frowny faces and
not smiley faces.
Finally we can eliminate the 2nd answer choice because we should have a smiley face
in the upper-right corner (where there was a down arrow in the left frame), and a sun
in the upper-left and lower-right corners (where in the left frame we had up arrows),
but we have the opposite.
We are left with the 4th choice as the only correct answer.

104. Explanation
The correct answer is 1.

There are two changes that occur between the first two figures in the first part of the
analogy: the figure rotates clockwise 90°, and the color changes from black to
white.

104
The same changes must apply to the third figure to complete the analogy.

This means that answer 1 is the correct answer.

105. Explanation
The correct answer is 3.

The relationship between the first two figures is as follows: from the first frame to the
second frame, all the circles become rectangles, and one of the small rectangles
disappears.
The correct answer must have the same relationship with the third figure: it should be
a large rectangle with three small rectangles.

This means that 3 is the correct answer.

106. Explanation
The correct answer is 2.
The relationship between the first two figures is as follows:
 The bottom shape (trapezoid) in the first figure becomes the outermost
shape in the second figure;
 The middle shape (triangle) is rotated 90° clockwise and becomes
the innermost shape the second figure;
 The top shape (circle) becomes the middle shape in the second figure.
The correct answer should have the same relationship with the third figure.
The bottom shape (rectangle) in the third figure must become the outermost shape.
This means we can eliminate answers 4 and 5.
The middle shape (triangle) must be rotated 90° and become the innermost shape, and
the top shape (circle) must become the middle shape. Answers 1 and 3 can be
eliminated, since they do not have the triangle in the correct position, leaving 2 as the
correct answer.

107. Explanation
The correct answer is 4.

The relationship between the first two figures is as follows:

- The inner shape (triangle) in the first figure moves to fill in the missing piece of
the outer shape.
- The inner shape (triangle) is rotated 90° counterclockwise.
- The inner shape (triangle) changes from white to black.

The correct answer will have the same relationship with the second figure. Since the
inner shape must move to the outside of the outer shape, we can eliminate answers 1
and 5. Answers 2 and 3 can also be eliminated since the outer shape is not supposed to

105
rotate.

This leaves 4 as the correct answer.

108. Explanation
The correct answer is 5.

The second figure is the same as first figure with two changes: the figure
becomes shaded, and the figure is extended in the direction of the dark area. The
correct answer should have the same relationship with the third figure.

This means that 5 is the correct answer.

109. Explanation
The correct answer is 2.

The relationship between the first two figures is as follows: the shapes in the first
frame each become shapes with one less side, the black shape becomes white and
the white shape becomes black. The correct answer must have the same relationship
with the third figure. The correct answer will be a large black triangle with a small
white four-sided shape.

This means that 2 is the correct answer.

110. Explanation
The correct answer is 4.

The relationship between the first two figures is as follows: the shapes in the first
frame each become shapes with one more side, the black shape becomes white and
the white shape becomes black. The correct answer must have the same relationship
with the third figure. The correct answer will be a large white pentagon with a small
black four-sided shape.

This means that 4 is the correct answer.

111. Explanation
The correct answer is 1.

The relationship between the first and second figure is as follows: the shape on the
left disappears, and the remaining shape on the right turns black. The correct
answer must have the same relationship with the third figure. This means that the
triangle on the left will disappear, and the remaining rectangle on the right will remain
and turn black.

106
This means that 1 is the correct answer.

112. Explanation
The correct answer is 3.

In the top row, from left to right, the lower half of the figure is removed. The same
should happen in the bottom row.
Since in the left frame we have two grey circles on top and two black circles on the
bottom, in the right frame we should only have the grey circles on top. We can
eliminate all the answer choices with black circles on the bottom (the 1st, 2nd, 4th,
and 5th choices).

We are left with the 3rd choice, which is the only correct answer.

113. Explanation
The correct answer is 3.

Look closely at the top row from left to right:


In the left frame there is a circle with a vertical and a horizontal line inside. In the
next frame (on the right), the circle shrinks and, in the center, overlaps the lines.

The same pattern should occur in the bottom row:


In the left frame, there is a diamond with two diagonal lines. In the next frame, the
diamond should shrink and overlap the diagonal lines in the center of the frame.

Answer choice 1 is incorrect as the diamond is transparent and the lines can be seen.
Answer choice 2 is incorrect since the lines are no longer diagonal and the shape in
the middle is a circle instead of a diamond.
Answer choice 4 is incorrect since the lines are not diagonal and there is no shape in
the center.
Answer choices 5 is incorrect as it contains a black (not grey) diamond.

We are left with the 3rd choice, which is the only correct answer.

114. Explanation
The correct answer is 1.

Take a look at the top row. The order of the shapes in the left frame determines the
position of the shapes in the right frames.
 The top shape (the star) is positioned in the center,
 The middle shape (the circle) is positioned on the outside, and -
 The bottom shape (the triangle) is positioned in between the two objects.

The same rule will apply in the bottom row:

107
The circle sector will be in the center, so we can eliminate answer choices 4 and 5.
The plaque will be the outer shape, so we can eliminate answer choice 2.
The ring will be positioned in between both shapes. We can eliminate answer choice 3,
and are left with answer choice 1 as the correct answer.

115. Explanation
The correct answer is 3.
In the top row, from left to right:
 The figure has been sliced diagonally and only the top left side of it remains.
 The figure became thicker.
In the bottom row, we should have the same relationship.
Image 1 is incorrect because, even though the top left side remained, the figure
became thinner.
Image 2 and 4 are incorrect because their figures are not sliced diagonally.
Image 5 is incorrect because the top right side of the figure has remained.
Image 3 is the only figure that matches the pattern in the top row, therefore, it is the
correct answer.

116. Explanation
The correct answer is 1.

The relationship between the first two figures is as follows: the second figure
represents all of the lines which are shared by the two shapes in the first figure.
Each of the two shapes in the first figure has a diagonal line which is not shared with
the other. These lines disappear in the second figure.
The correct answer must have the same relationship with the third figure.

This means that 1 is the correct answer.

117. Explanation
The correct answer is 4.

The relationship between the first two figures is as follows: the shapes in the first
frame each become shapes with one more side, the black shape becomes white and
the white shape becomes black. The correct answer must have the same relationship
with the third figure. The correct answer will be a large white pentagon with a small
black four-sided shape.

This means that 4 is the correct answer.

118. Explanation
The correct answer is 1.

108
In this question, the pictures consist of a varying number of small triangles inside a
large varying figure. There are two different patterns, one for the large figures and one
for the small triangles.
 The pattern for the large figures is that they move along one space, to the left
as you look down the rows, or upwards if you look across the columns.
 Notice that each row and each column include exactly one of each figure: one
heart, one circle, and one diamond-shaped square.
The pattern for the small triangles is as follows:
 across the rows from left to right, or down the columns from top to bottom, in
the third picture (the right or the bottom), the number of triangles is the sum of
the number of triangles in the first two pictures.
 That means that the missing picture should include four triangles.
o Based on the pattern of the large figure, the missing picture should be a large
circle, thus eliminating answers 2 and 4.
o Based on the small triangles pattern, the number of triangles in the missing
picture should be four, thus eliminating answers 3 and 5.
The only remaining possible answer is the 1st choice, and therefore the correct answer
as it matches the two patterns.

119. Explanation
The correct answer is 2.

In the top row, from left to right, we added a duplicate shape (another arrow that
was flipped 180 degrees) over the original shape and the overlapping area became
black.

In the bottom row we should have the same relationship. Since in the left frame we
have a square ring shape, in the right frame the shape should be doubled, so we can
eliminate the 1st answer choice. Further, only the overlapping areas should be black,
so we can also eliminate the 3rd, 4th and 5th choices.

We are left with the 2nd answer choice as the only correct answer.

120. Explanation
The correct answer is 4.

In the top row, every arrow in the left frame goes with a particular figure in the right
frame.
 Arrow pointing right = white heart
 Arrow pointing down = grey smiley face
 Arrow pointing up = grey sun

In the bottom row we should have the same relationship. We can eliminate the 3rd
answer choice because it contains a grey heart and not a white heart.

109
We can also eliminate the 1st and 5th choices because they contain frowny faces and
not smiley faces.
Finally we can eliminate the 2nd answer choice because we should have a smiley face
in the upper-right corner (where there was a down arrow in the left frame), and a sun
in the upper-left and lower-right corners (where in the left frame we had up arrows),
but we have the opposite.

We are left with the 4th choice as the only correct answer.

121. Explanation
The correct answer is 5.

Look carefully at the change in the top row from left to right.
The background changes from grey to white, and each circle changes to a
square with the shaded part switching from left to right, or right to left (but not up
and down).

In the bottom row we should have a similar relationship. We can easily eliminate the
2nd answer choice because it contains circles (not squares), and the 4th choice
because the background is grey (not white). We can also eliminate the 1st choice
because it changed in the opposite way than how it was supposed to (the black section
rotated from its position in the left frame and switched sides from its position in the
frame above, while it should have done the opposite). Finally, we can eliminate the
3rd answer choice because the black sections moved in different directions for each
shape, and didn't follow any specific rule.

We are left with the 5th choice as the only correct answer.

122. Explanation
The correct answer is 2.

In the top row, the outer shape in the left frame becomes the inner shape in the
right frame, while the inner shape in the left frame becomes the outer shape in the
right frame. The middle shape remains the same. Furthermore all the shapes swap
colors (from black to grey or from grey to black).

In the bottom row we should have the same relationship, as the outer shape in the left
frame is a hexagon, the inner shape in the right frame should also be a hexagon, we
can eliminate the 4th answer choice as the inner-most shape it contains is a pentagon
and not a hexagon. We can also eliminate the 3rd answer choice as it is missing the
middle shape. Finally we can eliminate the 1st and 5th choices because not all the
shapes swapped colors.

We are left with the 2nd choice as the only correct answer.

110
123. Explanation
The correct answer is 1.
 From left to right, the third image (on the right) in the first two rows is the
result of copying the lines that only appear in one of the first two images and
the black dots that appear in both images.
 Thus, the third image in the bottom row must be the result of combining the
lines that only appear in one of the first two images and the black dots that
appear in both images.
o Similarly, down the columns (top to bottom), the third image (on the bottom)
in the first two columns is the result of copying the lines that only appear in
one of the first two images and the black dots that appear in both images.
o Thus, the third image in the right column must be the result of combining the
lines that only appear in one of the first two images and the black dots that
appear in both images.
Hence, either by examining the rows or the columns, the same pattern holds, and by
using it, we can see that the only possible answer is the 1st choice, and therefore it is
the correct answer.

124. Explanation
The correct answer is 4.
In this question, each picture contains a square, divided into four parts, with an
additional blank smaller square in its middle. In each of the four parts (except in the
middle), there are rotated letter figures. The pattern is as follows:
 From left to right, each letter figure moves one spot clockwise (with its
orientation remains the same).
 Also, the small white blank square in the middle turns to grey.
The only answer that matches the description of the pattern of the top row, in
accordance with the left picture on the bottom row, is the 4th choice, and therefore it
is the correct answer.
o Answers 1 and 5 are incorrect because the small middle square is white
(should change to grey).
o Answer 2 is incorrect because the letter figures are rotated (and should remain
with their original orientation).
o Answer 3 is incorrect because the C letter figure is rotated (instead of
remaining in its original orientation like the other figures).

125. Explanation
The correct answer is 2.

In this matrix:
Across a row, the rightmost frame is the overlapping gray section of the previous two
frames.
Down a column, the gray part either increases by a ⅛ section in a clockwise manner
or decreases by a ⅛ section in a counter-clockwise manner.

111
Since in the third column the gray part is decreasing, we can eliminate the 1st, 3rd,
and 5th answer choices, as they all have gray sections that are the same or larger than
the previous (above) frame. We can also eliminate these choices because they do not
contain only the overlapping gray area of the previous two frames in the row. We can
also eliminate the 4th answer choice because the gray section is in the wrong position.

We are left with the 2nd answer choice as the only correct solution.

126. Explanation
The correct answer is 5.
In this question, in the top row, from left to right, the figure rotates 90°
counterclockwise and changes from white to black.
Therefore, for the second row, the figure on the left should rotate 90°
counterclockwise and change from black to white.
Hence, the 5th choice is the correct answer.

127. Explanation
The correct answer is 5.
In this question, the pattern for the top row is as follows:
 All three figures in the left picture are flipped upside down (180° rotation).
The rectangle seems to remain the same because a 180° rotation of a rectangle
does not change its orientation.
 The bottom-right figure in the left picture (the triangle) grows larger to
become the external figure in the right picture.
 The bottom-left figure in the left picture (the rectangle) remains the same size
and becomes inside the large triangle in the right picture.
 The top figure in the left picture (the pentagon) grows smaller, is also inside
the larger triangle, and is partially covering the bottom part of the rectangle.
For the bottom row, these same features should hold, so for the right picture:
o All figures in the left picture should be flipped upside down (180° rotation).
Notice that the rhombus and the circle remain the same.
o The bottom-right picture (the rhombus) should grow larger and be the external
figure.
o The bottom-left figure (the heart) should be flipped upside down and be inside
the large rhombus.
o The top figure (the circle) should grow smaller and be inside the rhombus,
partially covering the heart.
The only answer that matches all these criteria is the 5th choice and it is, therefore,
the correct answer.

112
128. Explanation
The correct answer is:

Across the row, the shapes remain the same.


In the top row, the large form is a circle, the medium form is a triangle, and the small
form is an arrow.
In the middle row, the large shape is a rounded square, the medium shape is a
parallelogram, and the small shape is an arrow.
In the bottom row, the large shape is a pentagon, the medium shape is a triangle, and
the small shape is an arrow.
We can already eliminate answer choice two as it contains an outer, rounded square
rather than a pentagon.
Additionally, across the rows, each large, medium, and small shape is presented in
three colors. Each shape does not appear in the same color twice. Therefore, in the
bottom row, the answer choice will contain a white pentagon, a gray triangle, and a
white arrow. We can further eliminate answer choice three as it has an image with the
wrong colors.
We are left with answer choices 1, 4, and 5. We can further see that the medium
shapes rotate 90 degrees clockwise across each row, and the small shapes (arrows)
rotate 45 degrees counterclockwise. We can also eliminate answer choices 4 and 5
and are left with answer 1.

129. Explanation
The correct answer is 5.
In the top row, the changes from left to right are of the number of small circles in each
of the four parts of the large circle and are as follows:
 The top-right and bottom-left parts remain unchanged.
 The top-left part decreases by two circles (there are three circles in the left
picture and one in the right picture).
 The bottom-right part decreases by one circle (two circles on the left picture
and one in the right picture).
Following the same pattern for the bottom row, the correct answer should have:
 One circle in the top-right part.
 One circle in the bottom-left part.
 No circles in the top-left part.
 No circles in the bottom-right part.
The only answer that matches this description is the 5th choice, and therefore it is the
correct answer.

113
130. Explanation
The correct answer is 5.
In this question, the pattern comprised of two elements, which are:
 Across the rows, from left to right, or down the columns, from top to bottom,
The third image (the right image in each row or the bottom image in each
column) is the result of combining the first and second images in the row or
column and then removing any overlapping lines.
 That means that the third image contains any lines that appear only in one of
the first two images, but not both.
 The little triangle that appears in some of the images changes sides (from left
to right, or vice versa) if it appears in one of the first two images but
disappears if it appears in both.
 Note that the pattern also might have been that the third image contains both
triangles if appear in both the first and the second images, but none of the
answer choices matches both elements of the pattern in that case.
o Thus, the third image in the bottom row, or the right column, must be the
result of combining the first and second images in the bottom row, or right
column, then removing any overlapping lines and erasing the little triangles.
Therefore, the only possible answer that matches these criteria is the 5th choice, and it
is the correct answer.

131. Explanation
The correct answer is 1.
In this question, the pattern for the top row is as follows:
 The left picture contains a straight line on the right and two figures, one on top
of the other.
 In the transition to the right, the bottom figure disappears, and the top figure is
duplicated and replaces the other bottom figure from the left picture.
Following this pattern, for the bottom row, since the left picture contains a straight
line on the left, the answer should have the line on the left.
o Answers 2 and 4 can therefore be eliminated as they contain a line on the right
instead of on the left.
In the right picture on the bottom row, based on the above pattern, the bottom figure
from the right picture should disappear and be replaced by a duplicate of the top
figure.
The only answer containing a picture that matches this description is the 1st choice,
and therefore it is the correct answer.

132. Explanation
The correct answer is 4.
 In this question, across the rows, from left to right, or down the columns, from
top to bottom, the third image (the right, or the bottom) in each of the first two
rows or columns is whatever is shared by the first two images in that row or
column.

114
 Thus, the third image in the bottom row or the right column must be whatever
is shared by the first two images in that row or column.
Examining either the rows or the columns, gets the same result, that the only possible
correct answer is the 4th choice:

133. Explanation
The correct answer is 1.
 In this question, each picture contains a large circle and four smaller figures
inside it.
 In the top row, from left to right, each small figure moves one place clockwise
and rotates 90° counterclockwise.
 Alternatively, the pattern can be that from left to right, each small figure
rotates 180°, and then the whole picture rotates 90° clockwise (or vice versa,
first the whole picture rotates 90° clockwise and then each small figure rotates
a 180° rotation).
Either way gets to the same result, and using this pattern for the bottom row, we can
see that the 1st choice is the only one matches the pattern, and therefore it is the
correct answer.

115
134. Explanation
The correct answer is 4.
In this question, in the top row, the pattern is as follows:
 from left to right, the bottom half of the hexagon remains the same, and the
top half changes in the following way: white triangles become grey, and grey
triangles become black.
 In the bottom row, the same pattern should lead to the correct answer, and
following it, the only answer that matches the description is the 4th choice,
and therefore it is the correct answer.

135. Explanation
The correct answer is 3.
In this question, the relationship between the pictures in the top row is as follows:
 The small internal figure on the left becomes the large external figure on the
right, and the large external figure on the left becomes the small internal figure
on the right.
 The vertical stripes of the internal figure on the left change to horizontal
stripes on the external figure on the right.
Following this pattern for the bottom row, the picture on the right should contain a
small white hexagon inside a large heart with vertical stripes (since the stripes on the
left are horizontal).
Therefore, the only possible answer is the 3rd choice, and it is the correct answer.
o Notice that since both figures on the top row are the same, the pattern could
have also been that the figures do not change and only the fill changes in the
transition. In such a case, the answer should have included a small white heart
inside a large striped hexagon. Such a picture is the one on the 2nd answer
choice; however, notice that the stripes are the same as in the left picture
(horizontal) when they are supposed to have changed to vertical, and therefore
this answer cannot be correct.

136. Explanation
The correct answer is 4.
In this question, all pictures consist of a varying number of figures, some are stars,
and some are squares.
There are two different patterns, one for the number of figures in each picture and the
other for the type of figure (stars or squares).
 The pattern for the type of figures in each picture is that they move along one
space, to the left as you look down the rows (top to bottom), or upwards as
you look across the columns (left to right).
 Also, notice that in each row and in each column, there are two pictures
comprised of stars and one picture comprised of squares.
The pattern for the number of figures in each picture is as follows:

116
 In each row and in each column, the number of figures in one of the pictures is
the sum of the number of figures in the other two pictures in that row or
column.
 The location of the picture that includes the sum number of the other two is
moving to the right if you look down the columns (top to bottom) or
downwards if you look across the rows (from left to right).
 Hence, the missing picture should include the sum of the figures in the other
two pictures in its row or column.
o Based on the figure type pattern, the missing picture should be comprised of
squares, and based on the figure number pattern, the missing picture should
include three figures.
Therefore, the correct answer, which is comprised of three squares, is the 4th choice.

137. Explanation
The correct answer is 3.

In the top row, from left to right, the lower half of the figure is removed. The same
should happen in the bottom row.
Since in the left frame we have two grey circles on top and two black circles on the
bottom, in the right frame we should only have the grey circles on top. We can
eliminate all the answer choices with black circles on the bottom (the 1st, 2nd, 4th,
and 5th choices).

We are left with the 3rd choice, which is the only correct answer.

138. Explanation
The correct answer is (B).
Identify the image that completes the pattern by examining how the series of shapes
change across the rows and down the columns within the matrix. The direction in
which you examine the shapes depends on where you can most easily visualize the
relationship between the shapes.
When you look at each row from left to right, notice the shape lose a line. When
moving from the leftmost box to the middle one, the shape loses its vertical line, and
when moving from the middle box to the rightmost box the shape loses the horizontal
line, leaving only the diagonal lines.
When moving from top to bottom in each column the shapes lose a line. When
moving from the top box to the middle box the shape loses the diagonals which
stretch from top-right to bottom-left, and when moving from the middle box to the
bottom the shape loses the other diagonal (which stretches from top-left to
bottom-right).
Answer (B) is the only answer which follows the pattern (the empty box), and
therefore it is the correct answer.

117
Alternative Solution: When moving either left to right or top to bottom, the shape
loses exactly one line. Therefore, the answer box must be empty to meet this
condition.

139. Explanation
The correct answer is 2.

The relationship between the first two figures is as follows: the shapes in the first
frame each become shapes with one less side, the black shape becomes white and
the white shape becomes black. The correct answer must have the same relationship
with the third figure. The correct answer will be a large black triangle with a small
white four-sided shape.

This means that 2 is the correct answer.

140. Explanation
The correct answer is 3.
In the top row, the changes from left to right are of the number of small circles in each
of the four parts of the figure and are as follows:
 The top-right part remains unchanged.
 The top-left part increases by one circle (one circle in the left picture and two
in the right).
 The bottom-left part decreases by one circle (two circles on the left picture and
one on the right).
 The bottom-right part increases by two circles (zero circles on the left and two
on the right).
Following the same pattern for the bottom row, the correct answer should have:
o Two circles in the top-right part
o Three circles in the top-left part
o Zero circles in the bottom-left part
o Four circles in the bottom-right part
The only answer that matches this description is the 3rd choice, and therefore it is the
correct answer.

141. Explanation
The correct answer is 1.

Take a look at the top row. The order of the shapes in the left frame determines the
position of the shapes in the right frames.
 The top shape (the star) is positioned in the center,
 The middle shape (the circle) is positioned on the outside, and -
 The bottom shape (the triangle) is positioned in between the two objects.

The same rule will apply in the bottom row:

118
The circle sector will be in the center, so we can eliminate answer choices 4 and 5.
The plaque will be the outer shape, so we can eliminate answer choice 2.
The ring will be positioned in between both shapes. We can eliminate answer choice 3,
and are left with answer choice 1 as the correct answer.

142. Explanation
The correct answer is 3.

In the top row we have two figures - how do they go together?


We can see that the two figures look similar, but:

• The figure on the left is flipped horizontally on the right.


• Also, the circle is now dark and has moved behind the shape.
Two new shapes have been added to the figure:
• A vertical line next to the dark circle, and -
• A white circle in the corner above it.

In the bottom row, we have a single figure, and we need to find a figure that will go
together with it in the same way that the two figures above go together.

We can immediately eliminate answer choice 4 because it belongs to the family of


figures in the top row, and not the bottom row.
We can also eliminate answer choice 2, because it is positioned in the same way as
the first shape, and is not flipped.
When we look at the other three answer choices, we see that the only one that matches
the pattern is answer choice 3.
Notice that answer choice 5 includes nearly all the changes we need, but it is a closed
shape and not an open one like the figures in the question.

Therefore the 3rd answer choice is the correct answer.

143 Explanation
The correct answer is 3.

In the top row we have two figures - how do they go together?


As we move from left to right:

• The white triangles change into black diamonds,


• The hearts rotate 180 degrees but remain gray,
• The number of circles increases by one, and -
• The circles change colors from black to white
In the bottom row, we have a single figure, and we need to find a figure that will go
together with it in the same way that the two figures above go together.

119
We can eliminate answer choice 2 as it contains a white diamond and not a black one.
In addition, this answer choice contains black circles instead of white ones.
We can also eliminate answer choice 1 as it contains a white heart and not a gray one.
We can eliminate answer choice 4 as it contains three circles instead of four.
Finally, we can eliminate answer choice 5 as it contains a heart that has not been
rotated.

We are left with answer choice 3, which is the correct answer.

144. Explanation
The correct answer is 1.

In this question you can see that on the top row:


- The left figure consists of five different shapes and all of them are white.
- When looking at the right figure you can see that the order of the shapes has been
reversed and the 2nd and 4th shapes changed their color to black.

Therefore, on the bottom row the order of the shapes in the right figure should be
(from left to right): lightning, ring-shaped circle, sun, triangle, and trapezoid.
Additionally, the ring-shaped circle and the triangle should be black as they are in the
second and fourth places.
The only answer that meets this criteria is answer 1.
Answer 2 is incorrect because the order of the shapes has not been reversed.
Answer 3 is incorrect because the 3rd and 5th shapes are black instead of the 2nd and
4th shapes.
Answer 4 is incorrect because the order of the shapes is incorrect. Tip: this answer
choice can be ruled out right away by noticing that the first shape on the left is a
triangle when it should be lightning.
Answer 5 is incorrect because the 1st, 3rd and 5th shapes are black instead of the 2nd
and 4th shapes.

145. Explanation
The correct answer is 2.
In this question, the pattern for the top row is, from left to right, as follows:
 All figures in the left picture are cut in half: the large external figure keeps its
bottom half; the internal bottom figure keeps its top half; and the top internal
figure keeps its bottom half.
 Also, in opposition to the left picture, there are no overlapping figures in the
right picture – all figure halves are exposed completely.
For the bottom row, these same features should hold, so the only possible answer is
the 2nd choice.

120
146. Explanation
The correct answer is 3.
In this question, each image contains a large square divided into nine parts, some
containing black or white plus signs. The pattern is as follows:
 Across the rows, from left to right, or down the columns, from top to bottom,
the third image (on the right or on the bottom) contains only plus signs that
appear in one of the other two images but not both.
 That means that the third image in each row or column contains the plus signs
that appear in the first image but do not appear in the second image, and the
plus signs that appear in the second image but not in the first one.
Either by examining the rows or the columns, using the pattern, we can see that the
only possible answer is the 3rd choice, and therefore it is the correct answer.

147. Explanation
The correct answer is 3.
In this question, the relationship between the top pictures is as follows:
 The small internal figure on the left becomes the large external figure on the
right, and the large external figure on the left becomes the small internal figure
on the right.
Since the two figures are the same (squares), it appears as if the pictures do not
change from left to right, and therefore it seems at first that the correct answer should
be a picture identical to the bottom-left picture. However, even though such a picture
would have also been a possible answer, it does not appear as an answer choice, and
therefore a different connection should be searched for.
The correct answer should therefore follow this pattern:
o the large external figure on the left picture (square) should become the small
internal one on the right picture, and the small internal figure on the left
picture (triangle) should become the large external figure on the right picture.
o The correct answer should be a small square within a large triangle.
Therefore, the 3rd choice is the correct answer.

148. Explanation
The correct answer is 3.

In the top row we have two figures - how do they go together?


We can see that the shapes themselves remain the same in both figures, but the
shading changes:
• The dark shading becomes white,
• The brick texture becomes dark, and -
• The white becomes brick-textured.

In the bottom row, we have a single figure, and we need to find a figure that will go
together with it in the same way that the two figures above go together.

121
We can immediately eliminate answer choice 2 because it belongs to the same family
as the figures in the upper row, and not the bottom row.
We can also eliminate answer choice 5 because the inner shape has changed.
When we look at the shading of the other 3 answer choices, the only one that follows
the same pattern as the figures above is answer choice 3.

Therefore, answer choice 3 is the correct answer.

149. Explanation
The correct answer is 5.
The pattern in the top row is as follows:
 From left to right, the large figure rotates 90° clockwise.
 The small figure on the bottom of the left picture that moves to the left of the
right picture due to the rotation keeps its orientation from the left picture,
while the other three small figures rotate together with the large figure
In the bottom row, the missing picture should be a 90° rotation of the left picture, with
the small bottom figure keeping its original orientation.
The only answer that matches this description is the 5th choice, and therefore it is the
correct answer.
o Answer 2 is incorrect since the picture rotation is counterclockwise instead of
clockwise.
o Answer 4 is incorrect since the left figure (which is the bottom figure of the
left picture) is rotated instead of keeping its original orientation.

150. Explanation
The correct answer is 2.
In this question, based on the top row, from left to right, the left picture characteristics
are:
 It consists of an X shape and three small figures: a circle in the centre, a
diamond-oriented square on the left, and a triangle on the bottom.
 The central circle is grey, and the two other shapes are white.
 The two white shapes partially cover the central circle.
In the transition to the right picture:
 The triangle moves to the right, and the square disappears.
 The central circle changes its colour to white, and the triangle changes its
colour to grey.
 The circle now covers the triangle instead of the other way round as it was on
the left.
On the bottom-left picture:
o The X shape and the central grey circle are exactly like the top-left picture.
o The bottom shape is also a circle (instead of the triangle in the top picture),
and the right shape is a pentagon (instead of the square in the top picture).
Following the top row pattern, the correct answer should follow these characteristics:
o White central circle.

122
o Grey circle on the right.
o The right grey circle should be partially covered by the central white circle.
The only answer that matches these criteria is the 2nd choice, and therefore it is the
correct answer.

151. Explanation
The correct answer is 2.
In this question, the relationship between the pictures in the top row is that from left
to right, the picture rotates 90° clockwise.
Therefore, the correct answer should be a 90° clockwise rotation of the bottom-left
picture:

Hence, the 2nd choice is the correct answer.


o Answer 1 is incorrect because the grey pentagon and arrow switched places.
o Answer 3 is incorrect because the white arrows on the top two corners are not
pointing in the correct directions.
o Answer 4 is incorrect because the picture is rotated counterclockwise instead
of clockwise.
o Answer 5 is incorrect because the picture is rotated 180° instead of 90°.

152. Explanation
The correct answer is 2.
Each of the pictures in this question is a square divided into 9 smaller squares. The
pattern is as follows:
 In the top row, from left to right, each of the squares that share a common line
with a square with a plus sign in it (linked to it, located beside, under, or over
it) becomes a square with a plus sign in it.
 The bottom row should follow this same pattern, and therefore, the squares
that should include a plus sign in them are the ones linked to the two squares
with a plus sign on the left picture.
The only answer that matches this description is the 2nd choice, and therefore it is the
correct answer.

153. Explanation
The correct answer is 1.

In the top row, from left to right, the figure changes as follows:

123
 A mirror image of the pentagon is added, forming an upside down pentagon
attached to the original one from below.
 The original pentagon changes its color from white to black.

In the bottom row, we should have the same relationship: A mirror image of the
triangle should be added to form an upside down triangle. This upside down triangle
should be attached to the original triangle from below. In addition, the original
triangle should change its color from white to black. Therefore, the 1st choice is the
correct answer.

The 5th choice is incorrect as the correct relationship between the figures does not
involve flipping the original figure from right to left.

154. Explanation
The correct answer is 5.

In the top row we have two figures - how do they go together?


As we move from left to right:
• The shapes on the right mirror the shapes on the left,
• The circle moves in front of the triangle,
• The background color changes from gray to black,
• The triangle changes from black to gray, and –
• The circle changes from gray to white.

In the bottom row we have a single figure, and we need to find a figure that will go
together with it in the same way that the two figures above go together.

Answer choice 2 can be eliminated because the background hasn’t changed to black.
Answer choice 3 can be eliminated because the circle remained at the back of the
triangle.
Answer choice 4 can be eliminated because the circle has move higher than in the
figure on the left.
Answer choice 1 can be eliminated because the square has moved to the back of the
triangle while it should have remained in place.

Answer choice 5 is the only one that follows all of the above rules and is the correct
answer.

155. Explanation
The correct answer is 3.

In the top row we have two figures - how do they go together?


We can see that the two figures look similar, but:

124
• On the right, the outer shape has received another side,
• The middle shape has been flipped vertically, and –
• The inner shape has become more rectangular.

In the bottom row we have a single figure, and we need to find a figure that will go
together with it in the same way that the two figures above go together.

We can immediately eliminate answer choice 5 because it belongs to the family of


figures in the top row, and not the bottom row.
We can also eliminate answer choice 2, because it has the same number of sides as the
first figure.
When we look at the other three answer choices, the only one that matches the pattern
is answer choice 3.
In answer choices 1 and 4 the figures are closed and not open.

Therefore the 3rd answer choice is the correct answer.

156. Explanation
The correct answer is 2.

The relationship between the first two figures is as follows: the second figure is the
same as the first figure, but rotated 90° counterclockwise, and with a triangle
added on top. The correct answer must have the same relationship with the third
figure.

This means that 2 is the correct answer.

157. Explanation
The correct answer is 4.

The relationship between the first two figures is as follows:

- The inner shape (triangle) in the first figure moves to fill in the missing piece of
the outer shape.
- The inner shape (triangle) is rotated 90° counterclockwise.
- The inner shape (triangle) changes from white to black.

The correct answer will have the same relationship with the second figure. Since the
inner shape must move to the outside of the outer shape, we can eliminate answers 1
and 5. Answers 2 and 3 can also be eliminated since the outer shape is not supposed to
rotate.

This leaves 4 as the correct answer.

125
158. Explanation
The correct answer is 5.

In the top row, from left to right, the shapes move to the center of the frame
and swap colors, and the bottom shape flips.
In the bottom row we should have the same relationship.

We can eliminate the 1st and 2nd answer choices because the shapes did not swap
colors. We can eliminate the 4th choice as the upper shape (the bar) did not move to
the center of the frame. Finally we can eliminate the 3rd choice because the lower
shape (the star) did not flip.

We are left with the 5th choice as the only correct answer.

159. Explanation
The correct answer is 4.
In this question, in the top row, the pattern from left to right is as follows:
 The larger figure grows smaller and moves to the bottom-left end of the
picture (and remains the same colour).
 The smaller figure grows larger and moves to the top-right end of the picture
(and remains the same colour).
 Both figures rotate a 180° rotation (although the diamond-shaped white square
appears to remain the same).
Hence, for the bottom row, the missing picture should contain a small grey partial
circle on the bottom-left end (a 180° rotation of the partial circle in the left picture)
and a large white full circle on the top-right end.
Therefore, the 4th choice is the correct answer.

160. Explanation
The correct answer is 3.

In this question, from left to right, we can see that:

 In the top row, shapes are inserted into the outer shape (triangle) to create the
figure on the right. The number of these inner shapes is the same as
the number of the outer shape’s sides: In the top row, the outer shape
has three sides, and thus three shapes are inserted. In the bottom row, the
outer shape has four sides, and thus four shapes should be inserted.
 In the top row, each of the newly inserted shapes has one side more than the
outer shape. The outer shape (triangle) has three sides, and each of the inserted
shapes has four sides (squares). In the bottom row, the outer shape (square)
has four sides, and thus each of the inserted shapes should have five sides
(pentagons).
 The inserted shapes are colored with blue.

126
The 3rd answer choice follows all the above conditions. Therefore, this is the correct
answer.

161. Explanation
The correct answer is 1.
In the top row, from left to right, the transition is as follows:
 The smallest internal figure on the left becomes the large external figure on
the right.
 The middle figure on the left becomes the small internal figure on the right.
The bottom row should follow this same pattern:
o since the left picture’s smallest internal figure is a heart, and the middle figure
is a square, the picture that matches the pattern should be a small square inside
a large heart.
Therefore, the 1st choice is the correct answer.

162. Explanation
The correct answer is 5.

In this question, from left to right, we can see that:

 In the top row, the left-top shape (the smiley face) is flipped over
upside-down but stays in its place. Thus, in the bottom row, the left-top
shape (the heart) should also be flipped over upside-down but stay in its place.
 In the top row, the left-bottom shape (the arrow) stays in its place and does
not change at all. Thus, in the bottom row, the left-bottom shape (the
trapezoid) should also stay in its place.
 In the top row, the right-top shape (the “L-shape”) and the right-bottom shape
(the triangle) switch places. Additionally, the new right-top shape (the
triangle) changes its color from white to black. Thus, in the bottom row, the
right-top shape (the star) and the right-bottom shape (the arc) should switch
places, and the new right-top shape (the arc) should change its color from
white to black.

The 5th answer choice follows all the above conditions. Therefore, this is the correct
answer.

163. Explanation
The correct answer is 3.

In the top row, from left to right, the number of black shapes swaps with
the number of circles (the two shapes and three circles in the left frame become three
shapes and two circles in the right frame).

In the bottom row we should have the same relationship. In the left frame we have

127
two circles, therefore in the right frame we should have only two shapes.
We can eliminate the 2nd and 5th answer choices.
In the left frame we have three shapes, therefore in the right frame we should have
three circles.
We can further eliminate the 1st and 4th choices.

We are left with the 3rd choice as the correct answer.

164. Explanation
The correct answer is 3.
In this question, the pattern for the top row is as follows:
 The left picture contains a diagonal line and two figures, one on the top-right
side of the line, and the other on the bottom-left side of the line.
 In the transition to the right picture, the top-right figure remains the same, and
the bottom-left figure is replaced by a 180° rotation duplicate of the top-right
figure.
Following this pattern, for the bottom row, based on the above pattern, the bottom-left
figure from the right picture should be replaced by a 180° rotation duplicate of the
top-right figure, while the top-right figure should remain the same.
The only picture that matches this description is the 3rd choice, and it is, therefore,
the correct answer.
o Answer 1 is incorrect because the bottom-left figure is not a 180° rotation
duplicate of the top-right figure but rather a diagonal mirror reflection of it.
o Answers 2 and 4 are incorrect because the diagonal line changes its direction
instead of remaining the same.
o Answer 5 is incorrect because the top-right figure is replaced with a 180°
rotation duplicate of the bottom figure instead of the other way round. The
top-right figure should remain exactly the same, and it has changed in this
choice.

165. Explanation
The correct answer is 2.

When we move from left to right across:


 In the top row, the color of the background of the frames changes from black
to white. The same change should occur in the bottom row.
 In the top row, the shapes at the bottom (the circle and the square) switch
sides. The same change should occur in the bottom row, and the triangle and
the rhombus should switch sides.
 In the top row, the shape on the bottom right (the circle) turns from grey to
black. In the bottom row the triangle should turn from gray to black.
 In the top row, the shape at the top (the triangle) rotates 180 degrees. The
same change should occur in the bottom row, and the ¾ circle should rotate
180 degrees.

128
The only answer choice which follows all rules is the 2nd.

166. Explanation
The correct answer is 2.
In this question, each picture consists of a large square divided into eight small
right-angled triangles. Some of the triangles are grey, and the rest are white.
In the top row, in the transition from the left picture to the right picture, the grey
colour changes from the internal diamond-shaped triangles to their identical matching
triangle outside it, as can be seen in the following illustration:

 as can be seen, each grey triangle on the left picture becomes white, and the
matching triangle that is oriented the same becomes grey.
Using this pattern for the bottom row, we can see that the only possible answer is
the 2nd choice, and it is, therefore, the correct answer:

o Although it may appear as if the grey colour moves to the triangle that shares
the diagonal line with each of the grey triangles, there is no answer choice that
matches a pattern as such.

167. Explanation
The correct answer is (A).
Identify the image that completes the pattern by examining how the series of shapes
change across the rows and down the columns within the matrix. The direction in
which you examine the shapes depends on where you can most easily visualize the
relationship between the shapes.
The outer shape in each row remains the same, however it rotates 90-degrees
clockwise. Therefore, the shape that will appear in the missing box will be a pentagon,
and it will be pointing upwards (in order to match the rotation pattern).
In addition, in each row and column, the shapes are filled with a different number of
black dots. The top two rows have shapes with one, two and three dots. As the third

129
row includes only shapes with two and three dots, the missing box must have a shape
with one dot.
The only answer that follows those conditions is answer (A).
Answers (B) and (C) contain the correct shape with the correct number of dots on the
inside (one). However, the pentagons they contain are not a 90-degree clockwise
rotation of the box to the left, and therefore they are incorrect.
Answers (D) and (E) have three and two dots inside, respectively. Since the column
and row already contain shapes with three and two dots, these answer choices are
incorrect and can also be ruled out.

168. Explanation
The correct answer is (B).
Identify the image that completes the pattern by examining how the series of shapes
change across the rows and down the columns within the matrix. The direction in
which you examine the shapes depends on where you can most easily visualize the
relationship between the shapes.
Look at the first row and try to find a connection between the triangles and the square.
Notice that the square on the right can be created by putting the two triangles together
and removing the diagonal line on the side they share:

The same connection can be applied to the second row as the two triangles can form
the parallelogram on the right. Therefore, the missing box will contain a shape
comprised of the two shapes on the other two boxes. In addition, the shapes in each
column have the same amount of edges which means the empty box must contain a
shape with four edges (like the square and the parallelogram).
The correct answer is (B) as the triangles in this row combined can form:

169. Explanation
The correct answer is (B).
Identify the image that completes the pattern by examining how the series of shapes
change across the rows and down the columns within the matrix. The direction in
which you examine the shapes depends on where you can most easily visualize the
relationship between the shapes.
Each box contains a different geometrical shape. Along the columns, the shapes
number of sides increases by one. For example, in the left column the shape changes
from a triangle to a square and then a pentagon; 3, 4, and 5 sides, respectively. This

130
rule is enough to answer the question because the answer must have one more side
than the octagon, making the correct answer (B).
Side note: You can also notice that the number of sides in the rows increases by 2.

170. Explanation
The correct answer is (E).
Identify the image that completes the pattern by examining how the series of shapes
change across the rows and down the columns within the matrix. The direction in
which you examine the shapes depends on where you can most easily visualize the
relationship between the shapes.
The left most box in each row is comprised of the other boxes in the same row.
Similarly, the top box in each column is comprised of the other two boxes in that
column.
Therefore, the empty box must contain a shape which, together with the adjacent
boxes, will complete both the shapes in the top box of its column and the leftmost box
of its row. Answer (E) is the correct answer.

Likewise, the shapes in the bottom row include objects that appear in only one of the
shapes in that column. If an object appears in more than one other shape, it will not
appear in the shape on the bottom row.

171. Explanation
The correct answer is (C).
Identify the image that completes the pattern by examining how the series of shapes
change across the rows and down the columns within the matrix. The direction in
which you examine the shapes depends on where you can most easily visualize the
relationship between the shapes.
Across each row the shape in the middle column is comprised from the objects which
appear in only one of the shapes adjacent to it (in the same row). Lines that appear in
both adjacent shapes are not included.
Similarly, in each column the shape in the middle row is comprised of the objects that
appear in only one of the shapes adjacent to it (in the same column). Lines that appear
in both adjacent shapes are not included.
Therefore, the correct answer is (C), as it is the only answer that completes the
pattern.
Alternatively, you could look at the column on the right and the bottom row. The
shapes in the column on the right include objects that appear in only one of the shapes
in the same row. If an object appears in more than one other shape, it will not appear
in the shape on the right column.

172. Explanation
The correct answer is (D).
Identify the image that completes the pattern by examining how the series of shapes
change across the rows and down the columns within the matrix. The direction in

131
which you examine the shapes depends on where you can most easily visualize the
relationship between the shapes.
Each row has the same shape with a different number of white dots. When moving
from left to right, the number of dots is divided by 2 (notice the number of dots is also
divided in each column when moving from top to bottom).
Therefore, the empty box must contain an arrow with exactly one dot (as two divided
by two is one), making answer (D) correct.
Side note: the location of the dots is random and only meant to distract from the
pattern.

173. Explanation
The correct answer is (E).
Identify the image that completes the pattern by examining how the series of shapes
change across the rows and down the columns within the matrix. The direction in
which you examine the shapes depends on where you can most easily visualize the
relationship between the shapes.
Across each row the white star moves from left to right. Down each column the white
star moves from up to down; in the top row it is positioned above the black stars, in
the middle row it is positioned in between them, and in the bottom row it is positioned
below them. The empty box is on the bottom row; therefore, the white star will be
below the black stars and it is also in on the right column, thus the star will appear on
the right. Therefore, the correct answer is (E).

174. Explanation
The correct answer is 4.
In every column and row of this matrix, there is one frame with a gray box, one frame
with a white box, and one frame with a black box. The gray box is the combination of
the other two boxes in that row and column (just like grey is a mix between the colors
black and white). That means that any circle that appears in a black or white box in
that row or column will appear in the grey box in the same spot and as the same color
circle. If a circle appears in the same spot in both the black and white boxes, and one
of the circles is white and the other is black, that circle will appear as a grey circle in
the grey box.

Since in the third row and in the third column, there is already both a black box and a
gray box, you should look for an answer choice with a white box. Thus, you can
eliminate answer choices (1), (2), and (3).

This leaves us with choices (4) and (5), so let us check to see if either one follows the
rule. You can start with either one and check it in the column or the row. Let's start
with (4) and check it in the row:

175. Explanation
The correct answer is (D).

132
Identify the image that completes the pattern by examining how the series of shapes
change across the rows and down the columns within the matrix. The direction in
which you examine the shapes depends on where you can most easily visualize the
relationship between the shapes.
Every box contains the same shapes – a square, a triangle and a circle, and one of
those shapes is patterned with diagonal lines. You can notice that each column has the
exact same shape, thus the empty box will contain a shape similar to its column.
Therefore, you can rule out answers (B), (C) and (E).
In addition, when moving across each row the patterned shape is either the inner,
middle or outer one: The top row has the outer shape colored, the colored shape in the
middle row is the one in the middle, and the bottom row has the innermost colored.
As the empty box is in the last row, the patterned shape will be the inner shape.
Therefore, the correct answer is (D).

176. Explanation
The correct answer is 4.
In this question, each image contains a large circle divided into three parts, some
containing black or white circles. The pattern is as follows:
 Across the rows, from left to right, or down the columns, from top to bottom,
the third image (on the right or on the bottom) contains only circles that appear
in one but not both previous pictures.
 Additionally, the circles’ colour changes from the colour of the previous
images (white turns to black and vice versa).
 For example, across the first row, the first image (on the left) contains a white
circle in the top-right part and a white circle in the bottom part of the large
circle; the second image (in the middle) contains a black circle on the top-left
part and another black circle in the bottom part; the third image (on the right)
contains a black circle on the top-right part (where the first image has a white
circle), a white circle in the top-left part (where the second image has a black
circle), and the bottom part of the third image is empty since both the first and
the second images in that row include circles.
Either by examining the rows or the columns, we can see using the pattern that the
only possible answer is the 4th choice, and therefore it is the correct answer.

177. Explanation
The correct answer is 1.

Every column contains two shapes. The left column contains a triangle and a circle,
the middle column contains a circle and a square, and the right column contains a
square and a triangle. Down the column, one shape increases in size while another
shape decreases in size. Take a look at the right column. The triangle becomes
progressively smaller as the square becomes progressively bigger. Therefore, the
answer choice will have a small triangle inside a large square. We can eliminate
choice 4, since it is missing a large square.

133
Additionally, in each row, each shape of the same size is a different color. In the
bottom row, there is a small, white circle and a small, black square. Therefore, the
answer choice will contain a small, gray triangle. We can eliminate answer choices 2
and 5. Furthermore, in the bottom row, there is a large, black triangle, and a large gray,
circle. Therefore, the answer choice will contain a large, white square. We can
eliminate answer choice 3 and are left with answer choice 1 as the correct answer.

178. Explanation
The correct answer is 5.
In this question, all images are comprised of a combination of an external figure and
an internal design:
 From left to right, the third image (on the right) in the first two rows is a
combination of the external figure of the second (middle) image in each row
combined with the internal design of the first image (left) in each row.
 Thus, the third image in the bottom row must be a combination of the external
figure in the second (middle) image in the bottom row with the internal design
in the first (left) image in the bottom row.
Alternatively, you can find that same pattern for the columns:
 from top to bottom, the third (bottom) image in the first two columns is a
combination of the external figure of the second (middle) image in each
column combined with the internal design of the first (top) image in each
column.
 Thus, the third image in the right column must be a combination of the
external shape of the second (middle) image in the right column with the
internal design of the first (top) image in the right column.
Using each of these methods will result in the 5th choice being the only possible
correct answer.

179. Explanation
The correct answer is 3.

As this question is composed of many elements, focus on each one separately. Across
the rows, notice that the white frame rotates clockwise around the grid. Therefore, in
the bottom row, the answer choice will contain a white frame in the top-right corner
of the grid. We can eliminate answer choices 2 and 5.

Across the rows, the black “L” shape rotates counterclockwise around the grid.
Therefore, in the bottom row, the black “L” shape will be located in the bottom-left
side of the grid. We can eliminate answer choice 4, as its "L" shape is located in the
wrong position. Additionally, across the rows, the black “L” shapes rotate vertically
from frame to frame. Therefore, we can eliminate answer choice 1 and we are left
with answer choice 3 as the correct answer.

134
180. Explanation
The correct answer is 3.

Take a look at the top row, going from left to right. The frames change in the
following way:

All the shapes from the left frame are rearranged as follows in the centre frame: the
bottom shape is placed within the (slightly enlarged) middle shape, which in turn is
placed within the (enlarged) top shape. Therefore, in the centre frame we have a small
L shape (from the bottom of the left frame), within a larger down arrow (from the
middle of the left frame), within an even larger square (from the top of the left frame).

The right frame is similar to the centre frame, with the following two differences:
1. The centre shape (in this case the L-shape) is elongated.
2. The middle shape has been replaced by the middle shape from the left frame
of the previous row (in this case the bottom row).
This pattern is repeated in each row.

We can eliminate choice 5 because the pentagon in the background has been flipped
and is not the same as in the centre frame. We can eliminate choice 1 because the
centre shape (the pie-shape) is not elongated. Finally, we can eliminate choices 2 and
4 as they should have a middle shape that is similar to the middle shape of the left
frame of the previous row, but they do not. (In choice 4 the middle shape has not
changed at all but remains a triangle, and in choice 2 the middle shape has changed to
that of the following [or in this case the top] row).

Therefore we are left with answer choice 3 as the only correct answer.

181. Explanation
The correct answer is 2.
In this question, each image contains a circle with various lines in it. There are two
different patterns, and using each of them separately will get you to the answer.
 The first pattern is across the rows, from left to right; the second image (the
middle one) in each row is a combination of the first (left) and the third (right)
images.
 Thus, the missing third image in the bottom row should include the missing
lines from the first image that appear on the second image.
o The alternative pattern is down the columns, from top to bottom; the third
(bottom) image in each column is a combination of the first two images (top
and middle).
o Thus, the missing third image should be a combination of the first and second
images in the right column.
The only answer that matches these patterns is the 2nd choice, and therefore it is the
correct answer.

135

You might also like